DESGLOSES COMENTADOS PEDIATRIA

August 5, 2017 | Author: Cristina Estébanez Prieto | Category: Cough, Hypothyroidism, Preterm Birth, Asthma, Medical Specialties
Share Embed Donate


Short Description

Download DESGLOSES COMENTADOS PEDIATRIA...

Description

Desgloses comentados

Pediatría T1 P141

P184

MIR 2011-2012

La enfermedad de las membranas hialinas o síndrome de distrés respiratorio del recién nacido prematuro es debida a la inmadurez pulmonar por déficit de surfactante pulmonar. Éste, fabricado por los neumocitos tipo II a partir de las 20-24 semanas de edad gestacional, es un agente tensioactivo que permite la apertura de los alveólos y, por tanto, un adecuado intercambio gaseoso. Está formado por proteínas, colesterol, fosfatidilcolina, esfingomielina y, fundamentalmente, por lecitina, cuya cantidad va aumentando según avanza el embarazo. La madurez pulmonar se alcanza finalmente a las 34-35 semanas de edad gestacional, de manera que por debajo de esta edad gestacional, la falta de surfactante provoca un aumento de la tensión superficial que determina el colapso alveolar y un shunt intrapulmonar (alveólos perfundidos, pero no ventilados). Clínicamente, observaremos un pretérmino con signos de dificultad respiratoria en las primeras horas de vida, con cianosis, mala respuesta al oxígeno, hipercapnia, acidosis respiratoria y crepitantes a la auscultación. En la radiografía de tórax observamos un parénquima retículo nodular fino, también denominado patrón en vidrio esmerilado, con broncograma aéreo y escaso volumen pulmonar. El tratamiento de elección es la administración de surfactante exógeno por vía endotraqueal, garantizando además un soporte respiratorio adecuado, evitando la hiperoxia, que es mal tolerada por el pretérmino (uno de los principales factores de riesgo para desarrollar retinopatía de prematuro). La principal complicación a largo plazo es la evolución a una displasia broncopulmonar. La administración de corticoides prenatales a la madre, ante la amenaza de parto prematuro, disminuye su incidencia y su gravedad.

P179

MIR 2008-2009

Neonatología

MIR 2008-2009

La cocaína no suele producir síndrome de abstinencia en el recién nacido, a diferencia de los opiáceos. Sin embargo, pueden aparecer complicaciones obstétricas, como el parto pretérmino, el desprendimiento prematuro de placenta, etc. Por otra parte, puede afectar directamente al feto, manifestándose como CIR, microcefalia, hemorragias intracraneales, anomalías digestivas o renales, muerte súbita, alteraciones conductuales o neurológicas. La respuesta correcta, por tanto, es la 3. Lo que produce la cocaína no es parto postérmino, sino todo lo contrario: pretérmino.

Hoy en día, es poco frecuente diagnosticar clínicamente un hipotiroidismo congénito debido a que, en su mayor parte, son diagnosticados con el cribado neonatal, que se realiza determinando los niveles de TSH obtenida entre los dos y los cinco días de vida (cuyos valores estarían elevados). Las manifestaciones del hipotiroidismo congénito tardan varias semanas en presentarse plenamente, estando totalmente establecidas hacia el 3º-6º mes de vida. Aparece una facies peculiar (cara tosca, párpados y labios tumefactos, nariz corta con base deprimida, hipertelorismo, boca semiabierta y macroglosia). Otras manifestaciones típicas son el retraso en la maduración ósea, la hernia umbilical, la ictericia prolongada, la letargia y el estreñimiento. El principal problema del hipotiroidismo congénito es su repercusión sobre el desarrollo intelectual. Éste sería más grave cuanto más se demore el diagnóstico (respuesta 3 correcta), por lo que es importante instaurar el tratamiento cuanto antes. Aunque la causa más frecuente de hipotiroidismo congénito es la disgenesia tiroidea, independientemente de cuál sea la causa exacta, ante cualquier caso de hipotiroidismo debemos apresurarnos a establecer el tratamiento sustitutivo lo antes posible.

P187

MIR 2008-2009

Un caso clínico complicado, donde resulta bastante difícil decidirse entre las opciones 2 y 5. Las otras son relativamente fáciles de descartar, por las razones siguientes: •• R1: no existen signos de hemorragia intraventricular, que normalmente cursaría con bradicardia, hipoventilación, convulsiones, hipotonía y fontanelas a tensión (sólo coincide la hipotonía, que es un dato inespecífico). •• R3: la fenilcetonuria es una enfermedad metabólica en la que se produce una acumulación anormal de fenilalanina. Se manifiesta varias semanas después del nacimiento. La mayoría de estos enfermos muestran piel clara, ojos azules y color claro del pelo (no puede sintetizarse la melanina, ya que la fenilalanina es un precursor suyo y no puede metabolizarse en el hígado). Es típico el retraso mental por acúmulo de fenilalanina en plasma hasta 30 veces por encima del nivel normal. •• R4: la enfermedad de la membrana hialina produciría una imagen de condensación pulmonar, con broncograma aéreo y habría alteraciones en la auscultación pulmonar (en este caso, nos dicen que ventila sin problemas). La verdadera dificultad estaría en distinguir la hipoglucemia de la sepsis neonatal. Al ser hijo de madre diabética tiene un claro factor de riesgo para padecer una hipoglucemia. Sin embargo, al ser un recién nacido pretérmino, también lo tiene para sepsis. Por tanto, debemos guiarnos por las manifestaciones clínicas.

Desgloses comentados

497

Pediatría

La presencia de acrocianosis y malestar general, así como el color terroso, podrían aparecer en ambos casos. Sin embargo, cuando se trata de una hipoglucemia, lo habitual sería la aparición de temblores o algún otro tipo de clínica neurológica (irritabilidad, convulsiones) que en este caso faltan. Como en esta ocasión predominan las manifestaciones inespecíficas, sin foco infeccioso claramente identificable, resulta más probable la respuesta 2 que la 5.

P185

MIR 2007-2008

Pregunta asequible acerca del estudio de infecciones connatales en el recién nacido, si bien hacía años que no aparecían en el MIR. Las tres primeras opciones tocan aspectos generales del manejo de estas infecciones. En todas, es útil el estudio serológico del RN (opción 2 correcta), especialmente de la IgM específica al germen, ya que es de síntesis fetal (recuerda que es pentamérica y que por sus dimensiones no cruza placenta). Puedes repasar todo lo comentado en la siguiente tabla. P185 (MIR 07-08) Infecciones connatales

El estudio placentario (opción 1 correcta), a nivel histológico, puede mostrar datos como amnionitis, infartos, hipervascularización, abruptio… E incluso puede cultivarse. Muchas veces, la clínica no es específica y se manifiesta como CIR, citopenias, ictericia precoz, e incluso de forma asintomática (opciones 3 y 4 correctas). Este estado asintomático puede acarrear secuelas irreversibles. En concreto, la forma de presentación más frecuente de la toxoplasmosis es una coriorretinitis silente que puede abocar a la ceguera en la edad adulta. Por ello, todos los recién nacidos infectados han de ser tratados con sulfadiacina, pirimetamina y ácido folínico durante el primer año de vida (opción 5 falsa). Asimismo, fíjate que las respuestas 1, 3 y 4 tienen un puede. En Medicina, ya sabes que nada es seguro. Es muy difícil que unas opciones tan poco categóricas resulten falsas.

P181

MIR 2005-2006

Pregunta de dificultad moderada acerca del retraso en la eliminación del meconio. En condiciones normales, el RN expulsa el meconio en las primeras 24-36 horas de vida. Si la eliminación de meconio no se produce, se acumula una masa compacta a escala anorrectal que impide la expulsión. Son causas de retraso en la eliminación de meconio: 1. El síndrome de colon izquierdo hipoplásico (frecuente en hijos de madre diabética). 2. Fibrosis quística. 3. Aganglionosis rectal. 4. Drogadicción materna. 5. Prematuridad. 6. Tratamiento con sulfato de magnesio de la preeclampsia materna. El hipotiroidismo congénito no cursa en principio con retraso en la eliminación del meconio, porque la clínica de instauración es tardía, estando completamente establecida a los tres o seis meses de edad. Son datos sugestivos: una facies peculiar (cara tosca con párpados y labios tumefactos, nariz corta con base deprimida, hipertelorismo con boca abierta y macroglosia), estreñimiento, hernia umbilical, ictericia prolongada, letargia y un retraso en la maduración ósea. Sin embargo, hoy en día el diagnóstico clínico es poco corriente gracias a las pruebas de screening precoz mediante la determinación de la TSH en muestra de sangre obtenida a las 48 h y a los cinco días de vida de todo RN. Unos niveles de TSH menores de 10 mcUI/ml son compatibles con la normalidad, mientras que si la TSH supera los 50 mcUI/ml el caso es compatible con un hipotiroidismo primario congénito y debe instaurarse precozmente tratamiento con levotiroxina con el objetivo de mejorar el pronóstico neurológico de estos pacientes. La causa más frecuente de hipotiroidismo congénito es la disgenesia tiroidea, que incluye tiroides aplásicos, hipoplásicos y ectópicos, siendo esta última la causa más común de hipotiroidismo congénito. En la primera plantilla de respuestas que dio el Ministerio se consideraba como respuesta correcta la opción 5; no obstante, posteriormente la pregunta fue anulada, ya que algunos autores consideran que el hipotiroidismo congénito puede estar también implicado en el retraso en la eliminación del meconio.

P168

MIR 2004-2005

Pregunta difícil sobre los efectos de los corticoides. La que no debe plantearte ningún tipo de duda es la 5, puesto que se indican con este fin, acelerar la síntesis pulmonar de surfactante. Además, reducen la incidencia de

498

Desgloses comentados

Pediatría

hemorragia de la matriz germinal (respuesta 3 correcta), de NEC (respuesta 4 correcta), de DAP y de neumotórax.El resto de las opciones no deben preocuparte.

P192 (MIR 04-05) Enterocolitis necrotizante. Radiología

La opción 1 es falsa, porque los corticoides únicamente aumentan el riesgo de infección cuando se emplean crónicamente, produciendo en tal caso inmunodepresión celular. Pero el uso puntual de corticoides, aunque sea una dosis relativamente alta, carece de este efecto.

P170

MIR 2004-2005

La incompatibilidad Rh requiere que una madre, Rh negativa, sea puesta en contacto previamente con sangre Rh positiva. En cambio, la incompatibilidad ABO puede afectar al primer hijo sin necesidad de sensibilización previa (respuesta 1 correcta). Esta circunstancia es más frecuente (respuesta 5 falsa), pero menos grave, que la incompatibilidad Rh, por lo que no se asocia a anemias fetales severas (respuestas 2 y 3 falsas). El test de Coombs indirecto es positivo y el directo también suele serlo, salvo en raras excepciones, de modo que la opción 4 también se podría considerar correcta. En consecuencia, estamos ante una pregunta que podría impugnarse.

P170 (MIR 04-05) Isoinmunización del recién nacido (RN)

P168

P192

MIR 2004-2005

La enterocolitis necrotizante suelen presentarla en el MIR como un cuadro de distensión abdominal y deposiciones sanguinolentas. Sin embargo, esta pregunta va un poco más allá. Observa que nos dan tres antecedentes que implicarían un mayor riesgo para esta enfermedad: prematuridad, bajo peso, necesidad de ventilación asistida... Si a esto añadimos un cuadro clínico compatible con obstrucción intestinal (estreñimiento, vómitos, distensión y dolor cólico), la opción que mejor encaja es la 1. De hecho, una de las complicaciones más habituales de esta entidad son las estenosis intestinales, que generan cuadros suboclusivos, como en este lactante que nos comentan.

MIR 2003-2004

Pregunta de dificultad moderada. Teniendo en cuenta la clínica descrita en el caso sería razonable dudar entre las dos últimas opciones, pero considerando los antecedentes expuestos, no queda opción a duda. La vitamina E es una vitamina liposoluble cuya deficiencia aparece unida a enfermedades malabsortivas graves y prolongadas: fibrosis quística, colestasis (y dentro de éstas la atresia de vías biliares), etc. La carencia nutricional no se ha descrito. Se manifiesta por un síndrome potencialmente reversible caracterizado por ataxia (cerebelosa y por afectación de cordones posteriores) y neuropatía periférica. Con respecto al resto de opciones, es interesante comentar la 5. Esta patología debuta en la infancia o adolescencia con ataxia progresiva de la marcha y, ya que su diagnóstico es eminentemente clínico (apoyándonos en los antecedentes familiares, EMG, etc.), es fundamental descartar previamente un déficit de vitamina E. Las opciones 1 y 3 son incorrectas, puesto que en el caso no se describe ninguna alteración del estado mental. El déficit de vitamina A cursa con ceguera nocturna, xeroftalmia, retraso del crecimiento, apatía, sequedad cutánea, etc. Como comentario final reseñar la importancia de la reposición periódica de vitaminas liposolubles en las colestasis neonatales.

Desgloses comentados

499

Pediatría

P191

MIR 2002-2003

Pregunta fácil sobre el distrés respiratorio del recién nacido. Mediante este caso clínico se nos exponen las principales características de lo que se conoce como: síndrome de Avery, pulmón humedo, taquipnea transitoria del recién nacido o síndrome de distrés respiratorio tipo II. •• Epidemiología: recién nacidos a término o postérmino por cesárea o por parto vaginal muy rápido. •• Patogenia: retraso en la absorción del líquido de los pulmones. •• Clínica: distrés respiratorio que se valora mediante el test de Silverman (regla mnemotécnica: DIRE QUE ATLETIC: DI-sociación tóraxabdomen; RE-tracción xifoidea; QUE-jido respiratorio; ALE-teo nasal y TI-raje). Mejora con la administración de oxígeno. •• Diagnóstico: Rx de tórax: líquido en las cisuras, hiperinsuflación, diafragmas aplanados, derrame pleural, no broncograma; GAB: normal. •• Tratamiento: atmósfera enriquecida en oxígeno, recuperación en uno o dos días. Por eso, la opción correcta es la 1. La opción 2 es el tratamiento de la enfermedad de membrana hialina o síndrome de distrés respiratorio tipo I (con la cual se podría plantear duda, porque es típico de recién nacidos pretérmino, pero su imagen radiológica típica es la afectación bibasal con infiltrado retículo-nodular y broncograma).

P192

MIR 2002-2003

Pregunta de dificultad media sobre un tema que es muy preguntado, como es la enterocolitis necrotizante. La enterocolitis necrotizante es una lesión isquémico-necrótica que suele afectar a íleon distal y colon proximal y que con frecuencia se acompaña de sepsis bacteriana (opción 1 incorrecta). •• Etiología: es típica de recién nacidos de muy bajo peso con enfermedades graves, policitemia, alimentación muy precoz y con elevados volúmenes, hipoxia... No obstante, parece que la lactancia materna puede ser un factor protector (también lo es de la retinopatía de la prematuridad) (opción 3 correcta). Se ha involucrado a diversos patógenos (E. coli, C. perfringens, rotavirus y S. epidermidis), pero en la mayoría de los casos no se detecta ningún germen responsable. (opción 2 incorrecta). •• Clínica: repentina distensión abdominal, retención gástrica y deposiciones sanguinolentas en la 2ª semana. •• Diagnóstico: laboratorio: leucopenia, neutropenia, trombopenia (opción 4 incorrecta), hiponatremia. Rx de abdomen: signos más precoces son el edema de asas y el asa fija; patrón en miga de pan, neumatosis intestinal (signo diagnóstico), neumoperitoneo (si hay perforación) y gas en vena porta. •• Tratamiento: dieta absoluta, fluidoterapia y descompresión nasogástrica; antibióticos; tratamiento quirúrgico si hay perforación, peritonitis o sepsis refractaria al tratamiento médico.

P177

MIR 2001-2002

Existen múltiples causas de dificultad respiratoria en el neonato. •• La taquipnea transitoria del recién nacido se produce por un retraso en la absorción del líquido de los pulmones fetales. Aparece en RNT o RNPT nacidos por cesárea o parto vaginal rápido. Clínicamente, el neonato presenta distrés respiratorio leve-moderado, de inicio precoz y que mejora con pequeñas cantidades de oxígeno. La gasometría no suele mostrar alteraciones significativas y en la radiografía se observa un aumento de las marcas vasculares, líquido en las cisuras, aplanamiento de los diafrag-

500

Desgloses comentados

mas y, a veces, derrame pleural. No existe broncograma aéreo (respuesta 4 correcta). •• La enfermedad de la membrana hialina (EMH) se produce por un déficit de surfactante pulmonar, lo que provoca un aumento de la tensión superficial y colapso de los alveolos. Afecta, sobre todo, a RNPT (dado que los niveles de surfactante no alcanzan la madurez pulmonar hasta las 35 semanas de edad gestacional). Cursa con un distrés respiratorio precoz moderado-severo, con cianosis progresiva que con frecuencia no mejora con la oxigenoterapia. En la gasometría hay hipoxemia, hipercapnia y acidosis metabólica y/o respiratoria, y en la radiografía es típico (pese a que no patognomónico, pues una radiografía similar se puede ver en la sepsis neonatal) un infiltrado reticulogranular con broncograma aéreo. •• El síndrome de aspiración de meconio (SAM) es un cuadro típico de RN postérmino que ha padecido un sufrimiento fetal agudo. La gravedad clínica depende de lo espeso del meconio. El RN presenta distrés respiratorio y signos de atrapamiento aéreo, con hiperinsuflación torácica y riesgo de extravasación extrapulmonar de aire, dando como complicaciones neumotórax, neumomediastino o neumopericardio. El neumomediastino suele ser asintomático, apareciendo en algunos casos una distensión de las venas del cuello, por estar dificultado el retorno venoso. Los tonos cardíacos están atenuados. La radiografía típica del SAM es un infiltrado parcheado, de aspecto algodonoso, con diafragmas aplanados. A veces también neumotórax o neumomediastino. •• El síndrome de Wilson-Mikity es un cuadro clínico radiológicamente similar a la displasia broncopulmonar (cuadro producido al someter a neonatos a ciertos procedimientos terapéuticos, como la oxigenoterapia a altas concentraciones) que se describió en RNPT sin antecedentes de EMH.

P178

MIR 2001-2002

La meningitis en el recién nacido puede estar causada por bacterias, virus, hongos o protozoos. La meningitis puede presentarse dentro de una sepsis o como una infección focal. Las causas bacterianas más normales de meningitis neonatal son los estreptococos del grupo B, E. coli y Listeria. También pueden producir meningitis otros estreptococos, cepas no tipificables de H. influenzae y estafilococos, entre otros. P178 (MIR 01-02) Sepsis y meningitis neonatal

Pediatría

P177 (MIR 01-02) Distrés respiratorio

La duda razonable está entre las opciones 4 y 5, dado que ambos pueden causar meningitis neonatal. Sin embargo, el estafilococo es uno de los gérmenes implicados en la sepsis de origen nosocomial, sobre todo en neonatos que han estado cateterizados. El cuadro clínico consiste inicialmente en una sepsis sin foco, apareciendo con frecuencia, más tarde, focalidad meníngea (respuesta 4 incorrecta; respuesta 5 correcta). Las manifestaciones clínicas de la meningitis neonatal pueden ser inicialmente inespecíficas, con o sin signos neurológicos (letargia, fontanela abombada, convulsiones...). El diagnóstico se confirma mediante el examen del LCR y la identificación del microorganismo. El LCR se considera patológico si hay pleocitosis (más de 30 células/mm3, con polimorfonucleares mayor del 60 %), hiperproteinorraquia (mayor de 150 mg/100 ml) e hipoglucorraquia (glucosa menor de 30 mg/dl).El tratamiento de la meningitis del RN no ingresado se establece empíricamente con ampicilina más cefotaxima, y en el RN ingresado con vancomicina más ceftacidima +/– anfotericina B.

P185

MIR 2001-2002

El trasplante hepático representa el tratamiento convencional de las hepatopatías terminales de la infancia. La indicación más frecuente en los niños es la atresia biliar extrahepática que no responde a la portoente-

rostomía de Kasai. Las enfermedades metabólicas del hígado, entre ellas el déficit de alfa 1- antitripsina, que es la más usual, y la colestasis familiar le siguen en frecuencia. En los centros de trasplante se utilizan varios sistemas de puntuación para predecir la mortalidad por hepatopatía, pero los factores más destacados son la prolongación del tiempo de tromboplastina parcial, la ascitis, el aumento de los niveles de bilirrubina indirecta y la disminución del colesterol. Algunos trastornos, como la atresia biliar sin respuesta a la cirugía de Kasai, deben corregirse con trasplante antes de que aparezcan complicaciones tardías. El tratamiento previo al trasplante resulta decisivo para el éxito de la intervención. Los aspectos más relevantes comprenden la nutrición, las vitaminas, la vacunación y la atención sanitaria general del niño. El tratamiento médico también comprende el control de la hipertensión portal. El estado del paciente en el momento del trasplante, a excepción de los casos de coma profundo, no influye en la supervivencia, pero sí afecta al período de recuperación y a las complicaciones. El pronóstico para los supervivientes es muy favorable. El crecimiento mejora y los estigmas de hepatopatía crónica desaparecen. Repasa las indicaciones de trasplante con la siguietne tabla.

Desgloses comentados

501

Pediatría

P185 (MIR 01-02) Indicaciones del trasplante de hígado en la infancia

Indicaciones

Nº de casos

• Atresia biliar • Enfermedad metabólica del hígado - Déficit de α -1 antitripsina - Tirosinemia - Enfermedad de Wilson - Otras • Colestasis familiar • Insuficiencia hepática fulminante • Hipoplasia biliar • Hepatitis • Cirrosis idiopática • Hepatitis neonatal • Colangitis esclerosante • Fibrosis hepática congénita • Carcinoma • Miscelanea

145 42 27 7 4 4 19 16 13 12 7 5 3 2 2 4

Total

270

De Ziell BJ, Gartner JC, Malatack JJ, y cols.: liver transplantation in children: A pediatrician´s perspective. Pediatr Ann 20:691,1191.

T2 P152

Desarrollo y nutrición

MIR 2010-2011

Pregunta sobre el desarrollo psicomotor del niño. En Pediatría son típicas estas preguntas, son bastante difíciles y específicas, ya que los periodos que nos proponen son muy concretos, y existen diversos hitos del crecimiento y desarrollo que pueden solaparse en diferentes momentos del tiempo. La prueba más empleada es el test de Denver, que valora los aspectos personal, social, lenguaje y motricidad fina y grosera en los niños desde el nacimiento hasta los 6 años. La respuesta correcta es la opción 3. Corre, sube escaleras andando cogido de la mano, construye una torre de cuatro cubos, se sienta en sillas pequeñas, dice 10 palabras (de media), identifica una o más partes del cuerpo, come solo, pide ayuda en caso de necesitarla, etc.

P166

MIR 2009-2010

Una pregunta de dificultad media que se responde de forma directa a partir del Manual CTO. Tal como explicamos en el Capítulo 2 (Desarrollo y nutrición) de la Sección de Pediatría, el indicador más sensible para detectar precozmente este tipo de alteraciones es la velocidad de crecimiento (respuesta 2 correcta). Ésta es máxima durante los cuatro primeros años de la vida, produciéndose un segundo incremento (aunque menor) durante la pubertad, siendo más precoz en niñas que en niños.

502

Desgloses comentados

Los datos que hay que tener en cuenta para evaluar una talla baja son la velocidad de crecimiento y la correlación con la talla de los padres. En este caso clínico se trata de un niño con una velocidad de crecimiento normal. Por otra parte, la talla se puede considerar baja pero dentro de los límites normales y, sobre todo, acorde con la talla materna. Además, nos explican que en la rama paterna, existen antecedentes de pubertad retrasada. Por consiguiente, no está indicado utilizar hormona del crecimiento ni realizar estudios del eje hipotálamo-hipofisario. Tampoco cabría esperar lo que dice la respuesta 3 (de hecho, sería al contrario: edad ósea retrasada con respecto a la edad cronológica). En otras palabras, no estamos ante un caso en el que haya que pensar en patología de ningún tipo, al menos con estos datos, por lo que no precisa que tomemos medidas extraordinarias

P188

MIR 2008-2009

Una pregunta aparentemente difícil sobre el desarrollo psicomotor que resulta mucho más sencilla si la analizas con lógica. La duda razonable estaría entre las opciones 2 y 3. De hecho, en esta convocatoria, a la mayor parte de los opositores les “sonaba” la idea de los seis meses. La dificultad está en decidirse por una u otra. En el Manual CTO, la solución aparecía claramente, y citamos de forma textual: “6º mes: inicia la sedestación, que se completa a los 8 meses”. Por tanto, si la sedestación se inicia en el 6º mes, difícilmente habrá un 50% de los lactantes que ya puedan sentarse antes de ese momento. La respuesta correcta sería entonces la 3. Por otra parte, ante una duda como ésta, es posible que, en el momento del examen, no dispongamos de ningún argumento teórico y haya que arriesgar entre dos opciones. En tal caso, ante una pregunta cuyas cinco opciones son numéricas, la apuesta más segura sería el valor intermedio, ordenándolos todos de menor a mayor. En realidad, así es como muchos opositores ganaron el punto de esta pregunta.

P181

MIR 2007-2008

El crecimiento y desarrollo en pediatría es un tema complejo debido a su amplitud. Para el MIR, solamente es necesario conocer un pequeño resumen. Esta pregunta se anuló. Durante la primera semana de vida el recién nacido (RN) puede sufrir una pérdida de peso fisiológica en torno al 5-10% con respecto al nacimiento, resultado de la excreción del exceso del líquido extravascular y la ingesta escasa. Por eso, la opción 2 es falsa, puesto que no suele disminuir más del 10% (pérdida de peso patológica). A medida que la madre mejora en la técnica de la lactancia y el calostro es reemplazado por una leche más rica en grasas, el RN recupera peso, alrededor de 30 g/día durante el primer mes (opción 3 correcta). Así, al final del primer año de vida, el peso al nacimiento aproximadamente se triplica y la talla se incrementa un 50% con respecto al nacimiento (si al nacer mide 50 cm, al final del primer año mediría 75 cm). Por ello, la opción 1 sería también falsa, dado que la duplicación de la talla ocurre alrededor de los cuatro años de vida. Las opciones 4 y 5 son correctas y hacen referencia al desarrollo psicomotor. Como norma, la mayoría de los reflejos arcaicos desaparecen en torno a los cuatro meses de vida. No olvides que el desarrollo psicomotor sigue un patrón cefalocaudal, y uno de los primeros hitos que aparecen es la sonrisa social hacia el 1,5-2 meses de vida.

Pediatría

P190

MIR 2007-2008

Una pregunta relativamente sencilla sobre la talla baja, ya que el síndrome de Klinefelter produce precisamente lo contrario: talla alta, sobre todo a expensas de la mitad inferior del cuerpo. Para acertar esta pregunta, en realidad no era necesario conocer rarezas como las que enuncian las respuestas 3 y 4. El síndrome de Seckel, entre otras manifestaciones, ocasiona microcefalia, bajo peso al nacer y talla baja. El de Silver-Russell, que es causa de talla baja, también se asocia con malformaciones esqueléticas (asimetría) y alteraciones morfológicas del cráneo. Los síndromes de Down y Turner son más conocidos como causas de talla baja. Recuerda que, por el contrario, el síndrome de Klinefelter (XXY), igual que el síndrome XYY, produce talla alta.

P180

MIR 2006-2007

Con respecto al déficit de hormona de crecimiento en la infancia, los pacientes que aquejan dicho déficit muestran una velocidad de crecimiento inferior a la normal y su curva de crecimiento se desvía progresivamente del canal normal. Pero si el déficit es congénito, se manifiesta a partir de los 6-12 meses de edad, siendo la talla y el peso al nacer normales. Eso es debido a que en esta etapa de la vida el crecimiento no depende de GH, sino de la insulina. La mayoría responden al tratamiento con GH sintética con una aceleración de la velocidad de crecimiento hasta un límite normal o, incluso, por encima de lo normal.

P188

MIR 2005-2006

Pregunta fácil de un tema constante en el MIR: repásala porque no puedes permitirte fallarla. Se trata, simplemente, de valorar una analítica general en un paciente pediátrico. Únicamente hemos de determinar el tipo de deshidratación (isotónica, hipotónica o hipertónica) y el tipo de acidosis (respiratoria, metabólica o mixta). Para contestar a este tipo de preguntas basta con conocer los niveles normales de sodio, pCO2 y bicarbonato. En general, quédate con las cifras 35-45, puesto que te serán útiles para varios de estos niveles: •• Na: 135-145 mEq/l. •• pCO2: 35 - 45 mmHg (aprovecho para recordarte los niveles del potasio que son 3,5-5, pero para recordarlos utiliza 3,5-4,5). •• El pH normal: 7,35-7,45. •• El bicarbonato oscila alrededor de 25 (21-28), pero en el MIR te darán niveles suficientemente extremos como para que decidas si está alto, bajo o normal. Sabiendo esto veamos los datos que nos ofrecen para responder la pregunta: •• El Na es de 137, por tanto, está comprendido entre los valores normales: se trata entonces de una deshidratación isotónica. •• El pH es de 7,20, consecuentemente, el paciente se encuentra en acidosis. El tipo de acidosis viene determinada por los niveles de pCO2 y de HCO3. Si el paciente no ventila adecuadamente (un ejemplo clásico del MIR sería la intoxicación por sustancias sedantes), la pCO2 aumentará y hablaremos de acidosis respiratoria. Si el paciente está perdiendo HCO3 por alguna vía (el ejemplo típico sería el de la pregunta, con diarrea y vómitos; pero también existen causas renales como la acidosis tubular proximal o tipo II), éste disminuirá dando lugar a una acidosis conocida como metabólica. En el caso de nuestra pregunta tenemos varios datos para decir que se trata

de una acidosis metabólica: por un lado el HCO3 está bajo (11) y, por otro, la clínica de diarrea y vómitos. Fíjate que la pCO2 lejos de estar aumentada como correspondería a una acidosis respiratoria, está descendida (pCO2 de 25); esto se explica por la compensación que está realizando el pulmón eliminando CO2 del organismo para corregir de esta forma el pH (recuerda estos binomios: hipoventilación-acidosis/hiperventilación-alcalosis). P188 (MIR 05-06) Tipos de deshidratación ISOTÓNICA HIPOTÓNICA HIPERTÓNICA Pérdida

Agua = solutos

Solutos > agua

Agua > solutos

Osmolaridad 285 mosm/l

300 mosm/l

Na

150 mEq/l

130-150 mEq/l

• Mucosas secas • Pliegue + • Fontanela deprimida Clínica

P171

• Hipotensión • Oliguria

• Mucosas secas + • Pliegue ++ • Mucosas secas ++ • Fontanela deprimida + • Sed • Hipotensión + • NRL (hemorragia subdural) • Oliguria + • Convulsiones

MIR 2003-2004

Aunque sea aparentemente compleja, esta pregunta se puede contestar fácilmente conociendo la definición de talla baja o hipocrecimiento. Se habla de hipocrecimiento cuando la talla se encuentra por debajo del percentil 3 para la edad y sexo. La paciente del caso tiene una talla en el P10, con lo que en principio debemos asumir que su crecimiento es normal (opción 1 correcta). Los demás datos del enunciado no hacen sino apoyar este diagnóstico. La paciente es una niña de diez años que todavía no ha iniciado el desarrollo puberal (estadio 1 de Tanner). Se considera que en el período prepuberal inmediato la velocidad de crecimiento debe ser al menos de 4 cm/año (la de la paciente es de 5 y, por consiguiente, normal). Por otro lado, la talla genética de la paciente (determinada por la talla media de los progenitores) está en el P15, bastante próxima a su talla real. Finalmente, se asume que la edad ósea de un paciente es normal cuando se encuentra en un intervalo de +/– 1 año en torno a la edad cronológica.

P176

MIR 2003-2004

Pregunta relativamente fácil si se tienen en cuenta las nociones básicas de alimentación del lactante. Hemos de fijarnos en el matiz que dan a lactancia materna en la respuesta 2: prolongada. El RN tiene unos depósitos de hierro suficientes para los primeros meses de vida. Esto hace que la lactancia materna sea suficiente para evitar la aparición de anemia ferropénica en ese período. Pero cuando éstos empiezan a agotarse, a pesar de la buena absorción del hierro de la leche materna, éste debe aportarse de forma extra mediante alimentos enriquecidos (como fórmulas de continuación). También es conveniente en esta pregunta recordar que tras los primeros días de vida, la causa más común de anemia es el déficit de hierro.

Desgloses comentados

503

Pediatría

P197

MIR 2002-2003

Pregunta fácil sobre la alimentación complementaria del lactante. Sobre la alimentación complementaria lo más importante es que se debe iniciar a partir del 4º-6º mes; no antes porque el lactante todavía tiene problemas de deglución, supone una sobrecarga de solutos y el aparato digestivo todavía no está preparado; pero tampoco después, porque la leche sola ya no cubre las necesidades del lactante. Lo primero que se debe introducir son los cereales sin gluten y se debe esperar al 6º-8º mes para comenzar con los cereales con gluten (por eso las opciones 3 y 5 son falsas). En cuanto al resto de opciones, en la 1 ya hemos comentado que precisamente esos cereales son los que no tienen gluten; la opción 2 no se ha relacionado con la dermatitis atópica, su introducción precoz sí que se ha relacionado con formas graves de celíaca; la opción 4 nos habla de la hidrólisis mediada por la amilasa pancreática que no influye para nada sobre los cereales. Por eso fue anulada, ya que todas las opciones son falsas.

P190

MIR 2001-2002

La muerte es un proceso que, obviamente, el niño vive de manera diferente al adulto. Ante la separación definitiva (la muerte de un progenitor, por ejemplo) la mayoría de los niños preadolescentes no parecen sufrir el típico proceso de duelo. El duelo del niño puede estar enmascarado por una conducta que no es típica de los adultos. Muchos niños continúan sus actividades cotidianas y el principal mecanismo de defensa ante la catástrofe es la negación, tanto manifiesta como inconsciente, mantenida por el deseo y la esperanza mágicos de reaparición y de reencuentro. Algunos niños muestran sentimientos hostiles y de ira hacia el progenitor que sobrevive, y tienden a identificarse con el progenitor perdido y a idealizarlo, a veces con fantasías de reencuentro que acompañan a la negación. Otra posibilidad es que los niños demuestren un dolor considerable al morir uno de los padres, o tras un lapso de tiempo, una vez que el mecanismo de defensa de la negación deja de ser eficaz. Los niños menores de cinco años creen que la muerte es reversible, y posiblemente piensan que los muertos vuelven a la vida o en los fantasmas. En la fase siguiente, hasta los ocho o nueve años, la muerte se personifica, como por ejemplo "el Coco" que castiga y se venga. Sólo pasada esta edad el niño comprende la muerte de forma realista, como un proceso biológico final y universal (respuesta 3 correcta).

T3

Aparato respiratorio

El paciente de la pregunta cumple con varios criterios: tos seca espasmódica de dos semanas de evolución, que incluso le han provocado hemorragias subconjuntivales por el esfuerzo. El agente microbiológico principal es la Bordetella pertussis, si bien también se han aislado Bordetella parapertussis y Bordetella bronchiseptica, aunque en menor proporción. La tosferina es una de las enfermedades transmisibles más contagiosas, siendo los lactantes menores de 6 meses, los adolescentes (como el caso de la pregunta) y los adultos los más susceptibles de padecer la enfermedad. Estos últimos actúan como transmisores al lactante, donde provoca mayor morbilidad. Normalmente, la tos ferina del adulto cursa como un cuadro catarral inespecífico. Pese a que el paciente esté correctamente vacunado, se sabe que la inmunogenicidad de la vacuna es relativamente variable. La eficacia frente a la enfermedad clínica es del 70-90% durante 2-5 años, y va disminuyendo con el tiempo, no prolongándose más allá de 12 años tras la administración de la última dosis de vacuna. El paciente recibió la última dosis de tos ferina a los 6 años, con lo cual la inmunidad esperada es menor. Por todo ello, la respuesta correcta es la opción 4: un macrólido, como tratamiento más eficaz frente al género Bordetella. La opción 1 (asma) se descarta porque la auscultación es normal y no hay signos de dificultad respiratoria. Serían precisos más estudios complementarios (opción 2 falsa) como muestras para aislamiento de Bordetella en una muestra clínica. Habría que estudiar a los contactos (opción 3 falsa) para descartar contagios. Por último, la hemorragia conjuntival (opción 5) es un dato típico. Se debe a la hiperpresión endotorácica, secundaria a los accesos de tos, que se transmite hacia los vasos conjuntivales.

P190

Una pregunta laboriosa por la extensión del caso clínico pero, en realidad, sencilla y referente a un tema característico del Examen MIR. Aunque la definición de bronquiolitis habla de lactantes menores de dos años con su primer episodio de dificultad respiratoria, aparece más en los menores de seis meses, como sucede en esta pregunta. Es importante tener en cuenta los síntomas previos para su diagnóstico, porque en los lactantes de más edad, sobre todo con antecedentes familiares u otros factores de riesgo para hiperreactividad bronquial (dermatitis atópica, alergias alimentarias, tabaquismo pasivo), la bronquiolitis puede confundirse con el asma. Las crisis de broncospasmo, a diferencia de las bronquiolitis, suelen tener una aparición brusca; en la auscultación se escuchan fundamentalmente sibilancias (sin ruidos de secreciones), y lo más importante, las crisis se repiten con frecuencia. Además, en este caso nos mencionan un episodio catarral previo, que también respalda la respuesta 2.

P192 P173

MIR 2009-2010

La OMS define como caso clínico de tosferina a aquel sujeto con tos que dura al menos dos semanas, con uno o más de los siguientes síntomas: •• Accesos de tos paroxística. •• “Estertor” al inspirar (gallo). •• Vómitos postusivos y sin otra causa obvia.

504

Desgloses comentados

MIR 2008-2009

MIR 2008-2009

El cuadro que nos describen corresponde a una tos ferina. Esta enfermedad es una infección producida principalmente por Bordetella pertussis. Afecta preferentemente a menores de un año. Clínicamente produce un pródromo inespecífico, de tipo catarral, y luego aparece la fase que caracteriza a la enfermedad, que es la de tos paroxística. Origina accesos repentinos de tos, con series repetitivas de múltiples toses enérgicas, acompañándose después de un ruido inspiratorio que recibe el nombre de “gallo”, al pasar aire a través de una glotis cerrada. La tos, en ocasiones, se asocia con vómi-

Pediatría

tos (tos emetizante). Las complicaciones de la tos ferina son: •• Neumonía: la más frecuente. Suele ser por sobreinfección bacteriana secundaria. •• Convulsiones, cuya causa no está bien establecida. •• Otras: prolapso rectal, hemorragia subconjuntival, hernia umbilical, epistaxis. Estas complicaciones se deben a la intensa presión positiva intratorácica que se produce en relación con la tos. A pesar de tratarse de una infección bacteriana, es característica la presencia de linfocitosis en el hemograma, como podemos ver en este caso. El tratamiento antibiótico de la tos ferina es la eritromicina durante dos semanas.

P227

MIR 2007-2008

Una pregunta más sencilla de lo que parece, dado que se presta a la deducción lógica. Nos preguntan por el virus que no causa neumonías (o lo hace con escasa probabilidad). Entre las opciones que nos exponen, sería lógico pensar que el virus respiratorio sincitial pueda producir infecciones broncopulmonares (de hecho, como sabes, es causa de bronquiolitis). También sabemos que el virus Influenzae (gripe) ocasiona cuadros respiratorios, así que no es de extrañar que el parainfluenzae produzca cuadros parecidos. Siguiendo este razonamiento, la duda estaría entre el adenovirus y el rotavirus. Sobre ellos, deberías saber: •• Adenovirus: estos virus producen infecciones respiratorias en niños y reclutas (colectivos cerrados, como campamentos militares, colegios.) y otros muchos posibles cuadros (diarrea, conjuntivitis, faringitis…). •• Rotavirus: es la causa más común de diarrea de origen vírico en el lactante, pero es rarísimo que produzca infecciones respiratorias (respuesta 4).

P192

MIR 2006-2007

Con respecto a la fibrosis quística, el diagnóstico se basa en las manifestaciones clínicas típicas (respiratorias, digestivas y genitourinarias) o antecedentes de fibrosis quística en un hermano o prueba de cribado neonatal positivo, más datos de laboratorio de disfunción de CFTR (test del sudor positivo en dos resultados positivos en días distintos) o dos mutaciones de FQ en estudio genético o alteraciones en la diferencia de potencial nasal. P192 (MIR 06-07) Diagnóstico de fibrosis quística

P186

MIR 2005-2006

Pregunta de dificultad fácil-media en forma de caso clínico de bronquiolitis. La bronquiolitis es una enfermedad viral que cursa con obstrucción inflamatoria de las pequeñas vías aéreas. Se define como el primer episodio de dificultad respiratoria con sibilantes espiratorios que tiene lugar en un niño < 2 años (el niño de la pregunta tiene dos meses), con síntomas de infección de vía respiratoria de tipo vírica. El virus respiratorio sincitial (VRS) es el agente causal más frecuente. La fuente de infección suele ser familiar y la transmisión se produce por vía respiratoria. Los niños mayores y adultos no presentan dificultad respiratoria a pesar de la infección, dado que toleran mejor el edema bronquiolar. En efecto, la contribución de la vía aérea de pequeño calibre a la resistencia total es mayor en los lactantes, y durante una infección por el VRS se produce una obstrucción bronquiolar causada por edema, acúmulo de moco y detritus celulares, con el resultado final de disminución del radio de la vía aérea y aumento reflejo de la resistencia al paso de aire. Es frecuente el antecedente de infección respiratoria leve unos días antes (hace tres días tuvo mocos, tos, estornudos, etc.) para aparecer posteriormente tos, dificultad respiratoria e irritabilidad. Habitualmente no existe fiebre, pero puede haber febrícula. Debido a la taquipnea, el lactante puede manifestar rechazo del alimento. En la exploración encontramos signos de dificultad respiratoria como aleteo nasal, tiraje, etc. En la auscultación se oye una espiración alargada y sibilancias. La disminución significativa de los ruidos respiratorios indica obstrucción casi completa de las vías aéreas y constituye, por tanto, un signo de gravedad. El diagnóstico es habitualmente clínico apoyado en la radiología. Se asume que bronquiolitis es todo aquel primer episodio de dificultad respiratoria baja que sufre un lactante menor de dos años, por otro lado, sano. La radiología muestra hiperinsuflación pulmonar. Se puede demostrar el virus en secreciones nasofaríngeas por inmunofluorescencia y elevación de los títulos de anticuerpos en sangre o cultivo. Para el tratamiento es esencial que recuerdes que los broncodilatadores son de muy dudosa eficacia, puesto que la fisiopatología de la enfermedad no es la broncoconstricción (como ocurre en el asma). Lo que más les beneficia es la administración de oxígeno. La adrenalina inhalada puede ser útil al producir vasoconstricción y, por consiguiente, ayudar a disminuir el edema. La ribavirina inhalada se reserva para casos graves. En resumen, recuerda que no están indicados ni broncodilatadores, ni corticoides, ni antibióticos ni sedantes. P186 (MIR 05-06) Diagnóstico diferencial entre bronquiolitis y asma

Desgloses comentados

505

Pediatría

El principal diagnóstico diferencial de la bronquiolitis se debe realizar con el asma. El asma: •• Es raro en menores de un año. •• No hay infección previa. •• Suele existir historia familiar. •• Las crisis son repetidas (solamente se llama bronquiolitis al primer episodio de dificultad respiratoria de un lactante menor de dos años). •• El comienzo es brusco.

P190

MIR 2002-2003

Pregunta de dificultad media sobre la patología congénita del aparato respiratorio. No es un tema muy preguntado en el MIR, pero donde las características de los distintos tipos de cuadros son muy parecidas. La laringomalacia y la traqueomalacia son las causas más habituales de estridor congénito. El diagnóstico diferencial entre ambas se realiza por el tipo de estridor; así, un estridor inspiratorio es más típico de una afectación de vías altas (laríngeo), mientras que si es tanto inspiratorio como espiratorio es típico de afectación de vías intermedias (tráquea). La afectación de vías bajas (bronquios, bronquiolos) lo que suele producir son sibilancias espiratorias. P190 (MIR 02-03) Patología laríngea en el niño

En consecuencia, en este caso la opción correcta sería el tratamiento conservador, puesto que es el primer paso a realizar. Las opciones 1 y 4 son maniobras invasivas que sólo se utilizarían en casos más graves; la opción 2 hace referencia al tratamiento quirúrgico del reflujo gastroesofágico.

P195

MIR 2002-2003

Pregunta fácil sobre la fibrosis quística. Se trata de una pregunta directa sobre un diagnóstico ya dado, la fibrosis quística, y donde nos la describen clínicamente como enfermedad multisistémica caracterizada por obstrucción crónica de las vías respiratorias, siendo la principal causa de enfermedad pulmonar crónica grave en la infancia y también la responsable de la mayor parte de las insuficiencias pancreáticas exocrinas en las primeras etapas de la vida. Desde el punto de vista genético, es la enfermedad hereditaria más frecuente y letal de la raza blanca. Se hereda de forma autosómica recesiva, estando el gen localizado en el brazo largo del cromosoma 7 (una regla para recordarlo es: FQ, si damos la vuelta a la F nos queda 7q). Por eso, la opción incorrecta es la 3. El resto de opciones nos describen las características patogénicas de la enfermedad. Desde el punto de vista clínico, lo más preguntado suele ser la frecuencia de sobreinfecciones bacterianas; teniendo en cuenta que el germen más frecuente: P. aeruginosa, el más precoz: S. aureus y el más grave: B. cepacea. Asimismo, la insuficiencia respiratoria crónica lleva al desarrollo en estos pacientes de un cor pulmonale. Desde el punto de vista diagnóstico se requieren dos criterios: •• Manifestaciones clínicas típicas o antecedentes de FQ en un hermano o pruebas de cribado neonatal positivas, más algún dato de laboratorio de disfunción de CFTR. •• Test de sudor positivo; dos mutaciones conocidas de FQ en estudio genético; alteración en la diferencia de potencial nasal. Repasa la figura de la pregunta 192, MIR 06-07. El tratamiento es fundamentalmente sintomático.

T4 P135

El diagnóstico se establece por laringoscopia directa. En general, el proceso se resuelve espontáneamente y no precisa tratamiento específico. Si hay dificultades para la alimentación, puede ser necesaria la alimentación por sonda. Muy excepcionalmente se utiliza la traqueotomía.

506

Desgloses comentados

Aparato digestivo

MIR 2011-2012

Pregunta de dificultad media debido a ser un tema que hacía años no aparecía en el MIR, si bien con los contenidos del manual, abordable. Fíjate que te presentan un neonato con imposibilidad al paso de sonda nasogástrica en el paritorio. La sospecha de atresia esofágica se confirma si sigues leyendo el enunciado donde menciona “bolsón esofágico atrésico”. Para filiar ante qué tipo de atresia nos encontramos (figura), nos comentan que la neumatización intestinal es NORMAL. Con estos datos excluimos las fístulas tipo I: donde sólo existe atresia y al no haber fístula no puede haber paso del aire al intestino; tipo II: la fístula proximal permite el paso únicamente a vía respiratoria sin neumatización intestinal; y tipo V: donde no

Pediatría

existe bolsón esofágico atrésico. Nos quedaría la tipo III y la IV, la primera con una frecuencia del 87% y la tipo IV con frecuencia inferior al 1%. De todas maneras esta pregunta no requiere un conocimiento tan profundo, ya que la tipo III es la más frecuente con diferencia (87%), la que has de saber, ya que es la que con probabilidad te preguntarían (recuerda la regla: la aTRESia esofágica más frecuente es la tipo TRES). P135 (MIR 11-12) Atresia de esófago

un retraso en la evacuación del meconio, que suele precisar estimulaciones para realizar deposición, que probablemente presentaría un retraso ponderal y que en el tacto rectal se apreciaría la ampolla vacía y una hipertonía del esfínter anal. Sin embargo, nos encontramos ante un lactante que realiza deposiciones de consistencia blanda de forma espontánea sin requerir estimulación ni laxantes, por lo que probablemente se trate de un estreñimiento funcional. El hecho de que las realice cada 5-6 días puede considerarse un ritmo intestinal normal. Por tanto, las opciones que deberíamos valorar son la 4 y la 5, aunque nos faltarían datos para contestar una de las dos con certeza. A continuación, se exponen las principales diferencias entre el estreñimiento funcional y la enfermedad de Hirschsprung. P136 (MIR 11-12) Diferencias entre estreñimiento funcional y enfermedad de Hirschsprung ESTREÑIMIENTO FUNCIONAL

ENFERMEDAD DE HIRSCHSPRUNG

Inicio al nacimiento

Raro

Frecuente

Enterocolitis

No

Posible

Tamaño de las heces

Grandes

Acintadas o normales

Retraso ponderal

Raro

Frecuente

Ampolla rectal

Llena

Vacía

Incontinencia fecal

Frecuente

Rara

Tono del esfínter

Variable

Elevado

P153 La prioridad de la cirugía es evitar la neumopatía causada por las microaspiraciones de ácido gástrico por la fístula distal, cada vez que el neonato inspira ejerciendo presión negativa (opción 3 correcta). De hecho, hasta que el neonato esté estable y se opere, se coloca en decúbito prono con objeto de evitarlas. Fíjate que las opciones 1 (imposibilidad de deglutir saliva) y 4 (imposibilidad de alimentación enteral) vienen a ser lo mismo y, por ello, son descartables, además de que no suponen una prioridad quirúrgica. La opción 2 (malformaciones cardíacas frecuentemente asociadas) se excluye con el enunciado “tras evaluación diagnóstica que descarta otras anomalías...”. En la clínica, hay que tener presente la asociación VACTERL (malformaciones Vertebrales, Anorrectales, Cardíacas, Traqueales y Esofágicas, Renales y radiales -Limb en inglés-) presente en un 30-50% de las atresias esofágicas; de ahí que lo mencionen en el enunciado. Por último, recuerda que una de las complicaciones postoperatorias es el reflujo gastroesofágico (MIR 05-06, 191), por anomalías intrínsecas de la función esofágica. Otras como traqueomalacia, estenosis esofágica, fístula de la anastomosis o de la fístula traqueoesofágica son más infrecuentes.

P136

MIR 2011-2012

En el caso que nos comentan, nos falta información tanto de la anamnesis, como de la exploración física, por lo que la pregunta fue anulada por el Ministerio. Ante un niño con estreñimiento desde el nacimiento, debemos diferenciar dos causas principalmente: un estreñimiento funcional y una enfermedad de Hirschsprung. Si estuviéramos ante una enfermedad de Hirschsprung describirían a un paciente que presentó

MIR 2010-2011

Pregunta fácil sobre un tema típico, repetido en varias convocatorias. Este tipo de preguntas no se pueden fallar. El cuadro clínico corresponde a una estenosis hipertrófica del píloro: lactante varón que debuta en torno a los 20 días de vida con vómitos alimentarios (no biliosos), “a chorro” o proyectivos, que le provocan una alcalosis metabólica hipoclorémica (se pierde HCl), con tendencia a la hipopotasemia, un estado permanentemente hambriento e irritable, deshidratación y desnutrición. En la exploración física, se palpa en algunos casos la “oliva pilórica”. La técnica de elección para confirmar el diagnóstico es la ecografía abdominal (respuesta 3 correcta). Cuidado con la respuesta 1, la Rx simple de abdomen es útil en el diagnóstico diferencial, pudiendo ver una única burbuja tanto en la hipertrofia como en la atresia pilórica y una doble burbuja en la atresia duodenal, pero nos piden la prueba complementaria de elección. El tratamiento preoperatorio consiste en fluidoterapia i.v. para corregir las alteraciones hidroelectrolíticas, siendo el tratamiento curativo la piloromiotomía de Ramsted. Hay que tener presente en la patología digestiva pediátrica los diagnósticos diferenciales, en este caso con la atresia pilórica (vómitos no biliosos desde las primeras tomas), la atresia duodenal (vómitos biliosos precoces, regla de la D: duodenal, doble burbuja, síndrome de Down), el reflujo gastroesofágico (vómito atónico de contenido alimentario desde la primera semana de vida, la pHmetría, respuesta 2, es la prueba más

Desgloses comentados

507

Pediatría

sensible y específica pero no la primera a realizar) y con la insuficiencia suprarrenal congénita (vómitos desde el nacimiento con alcalosis metabólica hiperpotasémica). P153 (MIR 10-11) Diagnóstico diferencial de la patología digestiva pediátrica PATOLOGÍA

Tipo de vómito

Radiografía

Estenosis hipertrófica de píloro

NO bilioso

Distensión gástrica con escaso gas distal

Atresia de píloro

NO bilioso

Imagen de úNICA burbuja

Atresia de duodeno

BILIOSO

Imagen de DOBLE burbuja

P170

MIR 2009-2010

Un tema que, desde hace muchas convocatorias, no aparecía en el examen MIR (en concreto desde 1996). No obstante, la pregunta es bastante sencilla, ya que la clínica del paciente tiene poco que ver con la que produciría una fisura anal (respuesta 1 falsa). Definimos intolerancia alimentaria como aquella reacción adversa que aparece tras la ingesta de alimentos, que no está mediada por IgE. Los síntomas son exclusivamente digestivos: diarrea, rectorragia, vómitos, dolor abdominal, etc. La alergia alimentaria, a diferencia de la intolerancia, sí está mediada por IgE. Clínicamente, aparte de los síntomas digestivos, podremos verlos de tipo alérgico: urticaria, angioedema, broncospasmo e incluso anafilaxia, en los casos más graves. La leche de vaca es el alimento más frecuentemente implicado en los procesos de intolerancia y alergia alimentaria. La proteína más relacionada con estos fenómenos es la beta-lactoglobulina, seguida de la alfa-lactoalbúmina y la caseína. El diagnóstico es fundamentalmente clínico. Los síntomas aparecen cuando el niño toma leche de vaca y desaparecen cuando este alimento es excluido de la dieta. El pronóstico de esta entidad es favorable, ya que desaparece a lo largo de los primeros años de la vida. Observa que, en el niño que nos plantean, solamente existen síntomas digestivos, por lo que se trataría de una intolerancia. Por lo tanto, no estaría mediada por IgE, como dice la respuesta 3. De hecho, la intolerancia es más frecuente que la alergia. El tratamiento consiste en sustituir la fórmula normal por un hidrolizado de proteínas lácteas. Los síntomas alérgicos pueden precisar un manejo específico, como antihistamínicos e incluso adrenalina subcutánea, si hubiese angioedema. En casos seleccionados se ha planteado el uso de cromoglicato sódico.

P175

MIR 2009-2010

La enfermedad de Hirschprung o aganglionosis colónica es todo un clásico en las preguntas del MIR de Pediatría. Antecedentes típicos son estreñimiento desde el período neonatal (tapón meconial) y a lo largo del primer año de vida (esta paciente tiene 4 meses), debido a la peristalsis inadecuada del segmento agangliónico. Otros signos típicos serían la distensión abdominal (megacolon) debido a la retención fecal y ausencia de gas distal,

508

Desgloses comentados

donde generalmente asienta el tramo agangliónico. Entre las complicaciones probables nos encontramos la de la pregunta: obstrucción. Inicialmente se toman medidas de desobstrucción, hasta la realización de pruebas más específicas que confirmen la sospecha diagnóstica (enema, manometría y biopsia): opción 3.

P180

MIR 2008-2009

Caso clínico típico sobre una estenosis hipertrófica del píloro. El síntoma cardinal en esta enfermedad lo constituyen los vómitos. Éstos serán no biliosos, proyectivos, y aparecen después (o casi después) de las tomas. Suelen comenzar a partir de los 20 días de vida. Debido a la pérdida de hidrogeniones y cloruros que supone el vómito, tiende a producirse una alcalosis metabólica hipoclorémica. En este caso clínico, las pequeñas estrías sanguinolentas pueden ser debidas a la intensidad del vómito, que puede producir hiperpresión a nivel de los vasos de la mucosa esofágica, dañándose algunos y produciendo un leve sangrado. En la exploración podemos encontrar grados variables de deshidratación o desnutrición, así como ictericia o subictericia. En algunos casos, incluso es palpable la “oliva pilórica” a nivel epigástrico, bajo el reborde hepático. La técnica de elección para confirmar el diagnóstico es la ecografía abdominal (respuesta 4 correcta). Se detectará un grosor del músculo pilórico por encima de lo normal (> 4 mm), así como un aumento de la longitud global del canal pilórico.

P182

MIR 2008-2009

Una pregunta difícil sobre el diagnóstico de la enfermedad celíaca. Ante la sospecha de esta enfermedad, lo habitual es realizar una determinación de anticuerpos (recuerda que los más sensibles y específicos son los IgA-antitransglutaminasa). Si esta determinación resulta positiva, estaría indicado realizar una biopsia intestinal, que también podría plantearse ante una determinación negativa, pero con una firme sospecha clínica. Por tanto, el papel de los anticuerpos en el diagnóstico de esta enfermedad tiene valor fundamentalmente como cribado. La biopsia intestinal es una prueba necesaria para establecer el diagnóstico. Se realizaría por lo menos en una ocasión, estando el paciente consumiendo gluten. En este caso clínico ya tenemos una biopsia intestinal compatible con enfermedad celíaca (la duda estaría en la necesidad de realizarla por segunda vez). La confirmación de la recuperación histológica después de la retirada del gluten (segunda biopsia) se realizaría en casos dudosos. No obstante, en este caso lo tenemos bastante claro: buena evolución clínica después de retirar el gluten de la dieta y negativización de los anticuerpos, por lo que en principio bastaría continuar con la dieta.

P186

MIR 2007-2008

Pregunta con un caso clínico típico de reflujo gastroesofágico madurativo o fisiológico. La manifestación clínica más usual es la regurgitación en la mayoría de las tomas a las pocas semanas de vida (la mayoría empiezan ya en la primera semana). Dado que el paciente presenta un adecuado desarrollo pondoestatural, hablamos de un reflujo gastroesofágico fisiológico debido a un esfínter esofágico inferior incompetente por inmadurez (opción 4 correcta).

Pediatría

La estenosis pilórica es un cuadro mucho más agudo, con vómitos proyectivos no biliosos, que comienza en torno a las tres semanas de vida con riesgo de deshidratación para el neonato (opción 2 falsa). Tanto la opción 1 como la 3 hacen referencia a reacciones adversas frente a la proteína de la leche de vaca, y por ello no guardarían tan buena evolución: se afectarían el peso y la talla. En la intolerancia, es común el hallazgo de heces con sangre. La alergia puede provocar urticaria, angioedema, broncoespasmo, e incluso anafilaxia como forma grave.

P191

MIR 2005-2006

Se trata de una pregunta de elevada dificultad sobre un tema poco preguntado en el MIR. P191 (MIR 05-06) Atresia esofágica

Por último, la opción 5, el vólvulo intestinal, se presentaría como un cuadro obstructivo y clínica de peritonismo. No permitiría una evolución tan solapada en el tiempo (hablan de dos meses y medio), sino mucho más aguda.

P188

MIR 2007-2008

La intolerancia a disacáridos provoca que éstos no se absorban adecuadamente. Mediante un efecto osmótico, atraen agua hacia la luz intestinal, de manera que las heces se vuelven acuosas (diarrea postprandial, opción 1 correcta). A su vez, al ser fermentados por la flora saprofita del colon, las heces se vuelven ácidas (por lo que aparece secundariamente eritema perianal) y se acompañan de meteorismo, debido al gas generado en la fermentación; así pues, éstas se eliminan explosivamente. En esto se fundamenta la prueba diagnóstica del H2 espirado (opciones 2, 3 y 4 correctas). Las heces esteatorreicas (grisáceas, brillantes, adherentes, flotantes…) son propias de los cuadros clínicos en los que existe malabsorción de lípidos (opción 5 falsa). En Pediatría, no olvides nunca la celiaquía ni la fibrosis quística.

P183

MIR 2005-2006

Pregunta de dificultad fácil-media acerca del diagnóstico de laboratorio de la intolerancia-malabsorción de lactosa. La malabsorción de lactosa se debe a un déficit de lactasa, que puede ser congénito, pero que lo más común es que sea secundario a procesos que lesionan de forma difusa el epitelio intestinal (infecciones, enfermedad celíaca...). Los disacáridos no hidrolizados se acumulan en el intestino, donde las bacterias producen ácidos orgánicos e hidrógeno, lo que atrae agua a la luz intestinal y se genera diarrea osmótica. Clínicamente se manifiesta por heces espumosas, con pH bajo, ricas en azúcar y que suelen excoriar las nalgas. Se produce también distensión abdominal, borborigmos y ocasionalmente dolor cólico. Para el diagnóstico podemos usar: •• Detección de cuerpos reductores en heces mediante la prueba del clinitest que se cuantifica según cambio de color. Recuerda que la sacarosa no es un azúcar reductor y, por tanto, si se sospecha intolerancia a la sacarosa, previamente debe hidrolizarse con ácido clorhídrico. •• Determinación de pH fecal inferior a 5,6 o determinación de ácido láctico en heces. •• Actividad de disacaridasas medida en la biopsia intestinal disminuida. •• Test de H2 en aire espirado tras sobrecarga oral de lactosa. El test de la ureasa en la biopsia intestinal se usa en el diagnóstico de infección por H. pylori. Se trata de un método invasivo, puesto que se hace a partir de una biopsia de antro y cuerpo gástrico. Se basa en que la ureasa que origina la bacteria hidroliza la urea y cambia el color de un indicador, lo que significa que hay infección por H. pylori. Por consiguiente, nada tiene que ver con el déficit de lactasa.

La atresia y fístula traqueoesofágica ocurren con una incidencia 1/3.0004.500 RN vivos. Existen diversas clasificaciones, aunque la más aceptada es la clasificación de Ladd, que distingue cinco tipos: •• Tipo 1: atresia sin fístula. •• Tipo 2: fístula proximal y atresia distal. •• Tipo 3: atresia proximal y fístula distal. •• Tipo 4: doble fístula. •• Tipo 5: fístula sin atresia. La forma más frecuente es la III, con casi el 85% de los casos. •• Debemos sospechar esta patología ante: •• Existencia de polihidramnios. •• Imposibilidad para pasar sonda nasogástrica en el paritorio. •• Salivación excesiva. •• Cianosis y atragantamiento con las tomas. Si existe una fístula traqueoesofágica distal, aparecerá una distensión abdominal importante, mientras que si no existe fístula distal, el abdomen estará excavado. Las formas con fístula proximal cursan con aspiraciones masivas con la alimentación. La fístula sin atresia (en H) puede cursar de forma más larvada y manifestarse como neumonías recurrentes.

Desgloses comentados

509

Pediatría

El tratamiento ha de ser quirúrgico. Durante el preoperatorio: posición en decúbito prono, aspiración continua del bolsón esofágico y medidas generales. La corrección quirúrgica suele realizarse en dos tiempos.

P192 (MIR 05-06) Invaginación intestinal. Imagen anatómica

Tras la cirugía la complicación más frecuente será los trastornos de la motilidad esofágica y, en particular, el reflujo gastroesofágico. Con menos frecuencia aparecen traqueomalacia (respuesta 5), estenosis (respuesta 2) o refistulización (respuestas 1 y 3).

P192

MIR 2005-2006

Se trata de una pregunta clásica de Pediatría que no podemos fallar el día del examen. Nos piden el diagnóstico de presunción y nos dan todos los datos típicos de una invaginación intestinal, así que aprovecha esta pregunta para repasar bien este cuadro. La edad es típica, ya que la invaginación es la causa más frecuente de obstrucción intestinal entre los tres meses y los seis años, sobre todo en menores de dos años, siendo raro en período neonatal donde la causa más habitual de obstrucción intestinal es la enfermedad de Hirschprung. No olvides que ante una invaginación intestinal en mayores de seis años se han de descartar causas orgánicas como el linfoma intestinal. La clínica descrita también es característica: aparición brusca de dolor abdominal intenso tipo cólico, crisis de llanto, encogimiento de miembros inferiores y palidez cutánea. A medida que avanza el proceso, el niño se encuentra más débil, somnoliento y aletargado. En fases iniciales suelen aparecer vómitos. La deposición de color rojo oscuro es también un signo muy típico de la invaginación intestinal en las primeras horas de evolución y se conocen como "heces en jalea de grosella". En fases avanzadas puede cesar la evacuación de gas y heces y aparecer un estado parecido al shock. Recuerda de cara al MIR que la mejor forma de hacer el diagnóstico es la ecografía abdominal, que muestra la imagen en rosquilla, donut o diana en un corte transversal. Otras formas de hacer el diagnóstico son la Rx simple de abdomen o el enema opaco con el que observamos el signo del muelle enrollado. Una vez diagnosticada la invaginación debe procederse a su reducción hidrostática siempre que se cumplan dos condiciones: la invaginación no debe ser prolongada y no debe haber datos de perforación intestinal. Se podrá realizar de dos formas: •• Reducción con aire o suero guiada con ecografía: esta opción asocia menor riesgo de complicaciones (perforación). •• Reducción con enema de bario guiada con control radiológico: esta opción presenta diez veces más de riesgo de perforación. Si existen signos de perforación, shock, neumatosis intestinal o distensión abdominal de más de 48 h de evolución, es preferible la corrección quirúrgica. Por último, recuerda que la forma más frecuente de invaginación intestinal es la ileocólica.

510

Desgloses comentados

P180

MIR 2004-2005

El cuadro que nos comentan sugiere una hernia de Bochdalek. Este tipo de hernia diafragmática es el más corriente, especialmente en el lado izquierdo. A través de la hernia, se produce un paso de órganos abdominales a la cavidad torácica, que es lo que justifica las imágenes aéreas que nos describen en el hemitórax izquierdo (Rx típica) (respuesta 4 correcta). Quizá la opción más difícil de distinguir sería la malformación adenoidea quística, que genera una imagen parecida (pulmón en queso de Gruyère). Pero, en este caso, las imágenes aéreas serían bilaterales. P180 (MIR 04-05) Hernias diafragmáticas

Pediatría

P181

MIR 2004-2005

P185 (MIR 04-05) Patogenia de la enfermedad celíaca

Ten mucho cuidado con este tipo de preguntas. Es bastante frecuente que intenten confundirte entre el estreñimiento funcional y la enfermedad de Hirschprung. Recuerda que en el estreñimiento funcional sí que es bastante habitual la presencia de encopresis. Sin embargo, sería muy raro encontrarla en la enfermedad de Hirschprung (respuesta 2 falsa). La rectorragia leve puede estar justificada por el traumatismo de la mucosa rectal en relación con fenómenos como impactación y/o heces muy endurecidas. El resto de los datos que nos ofrecen sí aparecen en el estreñimiento funcional (ver tabla siguiente). P181 (MIR 04-05) Diagnóstico diferencial del estreñimiento

P185 (MIR 04-05) Diagnóstico de la enfermedad celíaca

P174 P185

MIR 2004-2005

La enfermedad celíaca tiene una base inmunopatológica (respuesta 1 verdadera), habitualmente asintomática durante un tiempo, con un período libre de enfermedad que puede durar años, hasta que aparecen las primeras manifestaciones clínicas (respuesta 2 cierta). Dentro del diagnóstico diferencial, entrarían otras enfermedades digestivas, como la giardiasis (respuesta 4 cierta), que afecta también a duodeno e íleon proximal, compartiendo síntomas como flatulencia, esteatorrea, molestias epigástricas inespecíficas, etc. El diagnóstico de enfermedad celíaca requiere una biopsia de la unión duodeno-yeyunal. La lesión histológica no es específica, ya que puede verse en otras enfermedades, pero es muy sugestiva: atrofia de vellosidades e hiperplasia de las criptas (respuesta 5 correcta). No obstante, existen anticuerpos que también nos dan una orientación diagnóstica. Los más específicos son los antitransglutaminasa tisular (respuesta 3 falsa). Repasa con los esquemas siguientes la patogenia y diagnóstico de la enfermedad celíaca.

MIR 2003-2004

Es una pregunta fácil sabiendo la clínica típica de la estenosis hipertrófica de píloro EHP. Nos muestran un neonato de 21 días que presenta vómitos postprandiales cada vez más frecuentes. En un niño de esta edad y con esta clínica nos debemos plantear el diagnóstico de EHP. No olvides que el vómito no es bilioso y que el niño queda hambriento e irritable tras el mismo. De entrada podemos descartar la respuesta 4, puesto que la EHP es de etiología desconocida, pese a que se ha relacionado con factores que predisponen a ello como la administración de prostaglandina E para mantener la permeabilidad del ductus. Si nos fijamos en la opción 2, los vómitos persistentes efectivamente generan alteraciones electrolíticas, pero no acidosis metabólica, sino alcalosis metabólica hipoclorémica. Si el problema de la EHP es una hiperplasia e hipertrofia de la musculatura lisa del antro gástrico y del duodeno, es lógico pensar que el tratamiento será quirúrgico: pilorotomía de Ramsted (respuesta 3 falsa). Quedan aún las opciones 1 y 5. Sabiendo que la EHP es más habitual en primogénitos varones de raza blanca en la tercera semana de vida, podemos afirmar que la 1 es cierta. Recuerda que el polihidramnios no se relaciona con la EHP.

Desgloses comentados

511

Pediatría

P174 (MIR 03-04) Estenosis hipertrófica de píloro. Imagen anatómica

Supone la causa más usual de obstrucción intestinal baja en el RN (1/5.000 RN vivos), con mayor incidencia en varones (respuesta 2 verdadera), y puede aparecer asociado a otras alteraciones como síndrome de Down, Lawrence Moon Bield, Waardenburg y defectos cardiovasculares. La clínica más típica en el período neonatal es el retraso en la eliminación del meconio. En su diagnóstico se emplea la radiografía simple, que muestra distensión de asas con ausencia de aire rectal, el enema opaco, en el que se aprecia un cambio brusco en el diámetro del colon entre el segmento afecto (estenosado) y porción sana (distendido) (respuesta 5 verdadera); con la manometría anorrectal se observa una contracción del esfínter anal interno ante el aumento de presión a ese nivel (respuesta 4 falsa), siendo lo normal una relajación de dicho esfínter. El diagnóstico definitivo lo da la biopsia.

P189

MIR 2002-2003

Pregunta fácil sobre un tema típico en el MIR: la estenosis hipertrófica del píloro.

P175

MIR 2003-2004

Nos enfrentamos a una pregunta accesible acerca de la enfermedad de Hirschprung, que podremos contestar fácilmente con conocimientos generales de su clínica típica. Como sabemos, es una enfermedad congénita (respuesta 1 verdadera) cuyo origen está en una inervación anómala del intestino, con ausencia de células ganglionares del plexo intramural (respuesta 3 verdadera) por una migración anómala de neuroblastos. P175 (MIR 03-04) Enfermedad de Hirschprung

Se trata de una pregunta directa sobre las características de esta enfermedad: •• Epídemiología: es más frecuente en varones (opción 1 correcta), sobre todo en primogénitos. •• Clínica: el síntoma principal son los vómitos, no biliosos (opción 3 incorrecta), proyectivos (opción 4 correcta), que comienzan a partir de los 20 días de vida (opción 2 correcta). Tras vomitar, el niño queda irritable y hambriento. También son típicas la alcalosis metabólica hipoclorémica (debida al vómito) y la ictericia (opción 5 correcta) (debida a la deshidratación). •• Exploración física: palpación de la oliva pilórica y ondas peristálticas gástricas. •• Diagnóstico: clínica, pH, iones (suficiente en el 60-80%), ecografía abdominal (de elección), Rx con bario (signo de la cuerda) y Rx simple (única burbuja). •• Diagnóstico diferencial: atresia pilórica (vómitos no biliosos desde las primeras tomas con imagen de única burbuja en la radiografía), atresia duodenal (vómitos biliosos precoces e imagen de doble burbuja en la Rx, regla de la D: duodenal, doble burbuja, síndrome de Down) y reflujo gastroesofágico (vómito atónico de contenido alimentario ya en la primera semana de vida). •• Tratamiento: preoperatorio: fluidoterapia i.v. para corregir las alteraciones hidroelectrolíticas y quirúrgico (piloromiotomía de Ramsted).

P196

MIR 2002-2003

Pregunta bastante sencilla sobre la enfermedad celíaca, un tema nada infrecuente en el MIR. Si bien la pregunta es en forma de caso clínico, es todo muy típico y claramente orientador hacia el diagnóstico de enfermedad celíaca. La opción falsa es sin duda la 5: el tratamiento de la enfermedad celíaca es la retirada total y definitiva (de por vida) del gluten de la dieta, no concibiéndose su retirada gradual. El resto de opciones nos recuerdan datos relevantes sobre esta enfermedad: necesidad de, al menos, una biopsia (clásicamente 3); la predisposición genética (agrupación familiar, asociación con ciertos HLA) y su heterogeneidad clínica (tanto en tiempo como formas de presentación). Repasa la clínica de la enfermedad celíaca con el dibujo siguiente.

512

Desgloses comentados

Pediatría

P196 (MIR 02-03) Enfermedad celíaca

cocitos ni eosinófilos, donde no hay perdida de peso, se trataría de una diarrea crónica inespecífica que afrontaremos tranquilizando a la familia, pues es un cuadro benigno.

P201

MIR 2002-2003

Es una pregunta bastante sencilla, de manera que no se puede dudar al contestarla. Forma parte de uno de los temas más importantes de la pediatría, el aparato digestivo. La aparición de dolor abdominal brusco intenso de tipo cólico, crisis de llanto, encogimiento de miembros inferiores y palidez cutánea, es típica de la invaginación intestinal, opción 4 correcta. La invaginación es la causa más frecuente de obstrucción intestinal entre los tres meses y los seis años, y la localización más común es a nivel ileocólica. Como aparece en este caso, es típico la presencia de heces con sangre roja fresca y moco "heces en jalea de grosella". En la exploración clínica aparece como una masa dolorosa, localizada en hipocondrio derecho. Para confirmar el diagnóstico realizaremos un enema o una ecografía, que es la prueba de elección, donde veremos la imagen en rosquilla o diana. Respecto al tratamiento, mencionar que se puede intentar reducción hidrostática, o si aparecen signos de perforación intestinal o recidiva se manejará quirúrgicamente. En el supuesto de que fuera una estenosis pilórica, la paciente presentaría vómitos no biliosos a las dos o tres semanas de vida, sin heces con productos patológicos.

P200

MIR 2002-2003

Es una pregunta que hace referencia a las diarreas crónicas y que tiene una dificultad media. Es un caso clínico donde se nos presenta a una niña que tras un cuadro de gastroenteritis mantiene un proceso diarreico crónico (por durar más de dos semanas), sin productos patológicos, con irritación del área perianal (que nos indica un pH bajo). Ante este cuadro hemos de sospechar un déficit de disacaridasas secundario a un proceso que ha lesionado de forma difusa el epitelio intestinal, siendo éste el origen más frecuente. Estos pacientes también manifiestan con frecuencia dolor abdominal, borborigmos y, ocasionalmente, dolor cólico. Para confirmar el cuadro podemos realizar un clinitest, test de hidrógeno en aire espirado o medición de pH. El tratamiento estriba en la reducción de la ingesta de los azúcares responsables de la intolerancia a niveles que sean soportables, en este caso de forma transitoria. Por eso les diremos a los padres que alimenten a su hija con fórmulas sin lactosa durante varias semanas para que se regenere el epitelio intestinal. Este proceso no requiere tratamiento antibiótico, de hecho podría empeorar el cuadro por agresión al epitelio dañado, tampoco requiere rehidratación oral, puesto que en ningún caso se nos menciona que presente pérdida de líquidos que justifiquen el tratamiento. Los hidrolizados de proteínas serían el tratamiento de elección si fuera un paciente que presentara malabsorción proteica, con pérdida de peso, diarrea, hipoproteinemia y edemas. Si la paciente mostrara un aumento del número de las deposiciones con moco, fibras vegetales sin digerir y granos de almidón, pero sin sangre, leu-

El resto de las patologías que aparecen se manifiestan de diferente manera, de modo que no plantean dudas de diagnóstico diferencial, ya que, por ejemplo, en la apendicitis aparecería un dolor periumbilical sin vómitos ni letargia, además de no ser frecuente a esta edad.

P180

MIR 2001-2002

El reflujo gastroesofágico (RGE) es un cuadro relativamente corriente en el lactante, debido a diversos factores favorecedores: porción infradiafragmática del esófago más corta, estómago ligeramente más horizontalizado, menor capacidad gástrica, postura en decúbito..., pero no es fisiológico (respuestas 1 y 4 incorrectas). El síntoma más frecuente es el vómito, que en la mayoría de los casos aparece en la primera semana de vida (recuerda que los vómitos de la estenosis hipertrófica de píloro suelen comenzar hacia la 3ª semana). Suele desaparecer hacia los dos años de vida. El RGE puede presentarse también como alguna complicación: •• Respiratoria: neumonía por aspiración, tos, sibilancias, etc. •• Digestiva: esofagitis con hemorragia digestiva, anemia ferropénica, dolor, disfagia, etc. •• Retraso ponderoestatural. •• Síndrome de Sandifer: opistótonos y posturas anómalas de la cabeza, en relación con el RGE, bien como respuesta al dolor o como mecanismo de protección de la vía aérea (respuesta 2 correcta). El diagnóstico suele realizarse por la clínica y por la respuesta al tratamiento. Sólo en las formas más graves o en ausencia de respuesta al tratamiento empírico se realizan otras pruebas diagnósticas, tales como esofagoscopia

Desgloses comentados

513

Pediatría

y biopsia, esofagograma con bario y pHmetría de 24 horas, siendo esta última el método más sensible (respuesta 3 incorrecta).

El tratamiento del RGE se basa en tres pilares: •• Medidas generales: espesamiento de las tomas, posición semiincorporada. •• Medidas farmacológicas: procinéticos 15-20 minutos antes de tres tomas, antiácidos o inhibidores de la bomba de protones si existe esofagitis. •• Cirugía: cuando no se observa mejoría con el tratamiento médico o aparecen complicaciones severas del RGE se procede a realizar una funduplicatura de Nissen (respuesta 5 incorrecta).

P186

MIR 2001-2002

La invaginación supone la causa más frecuente de obstrucción intestinal entre los tres meses y seis años de edad. Se produce cuando un segmento intestinal se introduce en otro segmento inmediatamente distal a él. La forma más usual de invaginación es la ileocólica y la ileoileocólica. La gran mayoría de los casos son de etiología desconocida. Un pequeño porcentaje son secundarios a procesos como las infecciones por adenovirus, divertículo de Meckel, pólipos y otros tumores, etc. Clínicamente, se manifiesta con dolor abdominal intenso de tipo cólico, encogimiento de miembros inferiores y palidez cutánea. En fases iniciales suelen aparecer vómitos. En las primeras horas de evolución puede haber eliminación de heces, pese a que si la invaginación progresa cesa la evacuación de gases y heces. Hasta un 60% de los niños afectados pueden presentar heces con sangre roja fresca y moco ("heces en jalea de grosella"). Si progresa, el niño puede entrar en un estado parecido al shock. El diagnóstico se establece por la clínica y exploración física (se palpa una masa alargada, dolorosa, localizada en hipocondrio-flanco derecho). En el enema opaco se observa un defecto de repleción al nivel de la cabeza de la invaginación, y en la radiografía simple de abdomen, un área de aumento de densidad en hemiabdomen derecho y de distensión de asas en el izquierdo. La prueba fundamental es la ecografía, donde es típica la imagen en rosquilla o donut. El tratamiento consiste en la reducción hidrostática o neumática bajo control ecográfico o fluoroscópico. Es preferible la corrección quirúrgica en caso de signos de perforación intestinal, shock, neumatosis intestinal o distensión abdominal de más de 48 horas de evolución.

T5 P151

Nefrología y urología

MIR 2010-2011

Caso clínico en el que nos presentan una niña de 10 años con exploración física, talla y TA normales, el análisis de orina tiene una densidad y un pH dentro de los valores normales, el único dato alterado es el número de hematíes por campo (normal 0-4). Puesto que no tiene ningún síntoma (análisis de orina de rutina), crece

514

Desgloses comentados

correctamente y no nos indican ninguna otra alteración en el sedimento, carecen de sentido las opciones 1, 2, 3 y 5, por tanto, la conducta inicial más adecuada sería repetir el sedimento en 15 días, ya que el aumento de hematíes podría ser debido, por ejemplo, a la realización de ejercicio físico intenso, algo muy común en esta edad.

P189

MIR 2007-2008

Hasta la edad de los dos o tres años, los lactantes presentan una fimosis fisiológica en un altísimo porcentaje de casos (respuesta 1 correcta). Por ello, es evidente que no todos deberán ser intervenidos quirúrgicamente. Las indicaciones para la circuncisión en menores de un año son: •• Que la fimosis produzca infecciones urinarias. •• Balanopostitis. •• Parafimosis de repetición. •• Fimosis puntiforme (la que deja un orificio prepucial mínimo, de forma que obstaculiza la libre emisión de orina). Es cierto que los corticoides pueden ser útiles en algunos casos y evitar la circuncisión, pero esto no puede entenderse como la regla general, de manera que la respuesta 5 es falsa.

P193

MIR 2002-2003

Se trata de una pregunta de dificultad alta sobre un tema que puede aparecer tanto en Pediatría como en Nefrología o Infecciosas. El síndrome hemolítico urémico, desde el punto de vista pediátrico, es la causa más frecuente de insuficiencia renal aguda en niños menores de cuatro años. Desde el punto de vista infeccioso destaca que casi siempre va precedido por una gastroenteritis enteroinvasiva por E. coli (si bien también se ha relacionado con Shigella, Salmonella y Campylobacter). Desde el punto de vista clínico la tríada típica: anemia hemolítica con esquistocitos, trombopenia con pruebas de coagulación normales e insuficiencia renal. El principal diagnóstico diferencial se establece con la trombosis de la vena renal que cursa con HTA + hematuria + trombopenia. El tratamiento médico de las manifestaciones hematológicas, junto con una diálisis peritoneal precoz, es el que mejores expectativas de recuperación renal ofrece, de tal manera que la función renal se recupera en el 90% de los casos y es una enfermedad que no recidiva. De esta manera las pistas que podemos encontrar en este caso clínico son: •• Cuadro febril con diarrea mucosanguinolenta. •• Orina hematúrica. •• Anemia. •• Trombopenia con normalidad de las pruebas de coagulación. Podemos descartar otras opciones como: •• Púrpura de Schönlein-Henoch: la simple presencia de trombopenia descarta esta opción, porque es una púrpura no trombopénica. Su manifestación cutánea no son las petequias sino la púrpura palpable. Igualmente suele asociar glomerulonefritis, artralgias y dolor abdominal. •• Glomerulonefritis aguda postinfecciosa: suele aparecer en relación con infección faríngea o cutánea por estreptococo del grupo A tras un período de latencia, no coincidiendo con la infección. Desde el punto de vista renal cursa en forma de síndrome nefrítico (hematuria, HTA y oliguria).

Pediatría

P198

MIR 2002-2003

Pregunta fácil sobre la infección del tracto urinario. Las características de la ITU en niños son muy similares a las del adulto. En general, es más genérico en el sexo femenino, la única excepción es el primer año de vida, siendo más frecuente en este caso en los varones (opción 1 incorrecta). La causa más usual de ITU es E. coli (opción 2 correcta). También podemos encontrar otros gramnegativos como Klebsiella, Proteus... Menos frecuentes son los grampositivos (enterococo, S. epidermidis), adenovirus (cistitis hemorrágica), hongos... La clínica en los niños es bastante inespecífica. Lo que nos puede diferenciar una ITU baja de una alta es la presencia de fiebre en esta última. Esta distinción es primordial, sobre todo a la hora del tratamiento: bajaantisépticos urinarios, trimetropín-sulfametoxazol o amoxicilina oral; altaaminoglucósidos, cefalosporinas de 3ª generación o amoxiclavulánico inicialmente por vía sistémica (opción 4 correcta). El diagnóstico definitivo nos lo da el cultivo. En cuanto a la evaluación posterior, la ecografía abdominal se indica siempre ante cualquier ITU en un niño (opción 3 correcta), mientras que la combinación de eco, cistouretrografía miccional (despistaje de reflujo vesicoureteral y disinergia vesical) y gammagrafía renal con DMSA (método más sensible para demostrar presencia de cicatrices) (opción 5 correcta) estaría indicada ante toda ITU febril, ITU en menores de cinco años o en niñas en edades posteriores que hayan presentado dos o más ITU.

P187

MIR 2001-2002

El caso clínico nos muestra un cuadro de escroto agudo. La torsión testicular es la causa más común de escroto agudo en menores de seis años, y el primer cuadro a considerar en la edad prepuberal o en el adolescente con dolor escrotal agudo y tumefacción testicular.

Clínicamente, la torsión testicular cursa con dolor agudo y tumefacción del escroto, exigiendo diagnóstico diferencial con la orquiepididimitis. También hay que hacer el diagnóstico diferencial con la torsión del hidátide. Ésta es más frecuente entre los siete y los 12 años, suele producir menor dolor y tumefacción, y ocasionalmente se observa una zona azulada por encima del testículo con sensibilidad aumentada a la palpación. El tratamiento de la torsión testicular es la cirugía inmediata (respuesta 5 correcta). El procedimiento depende del tiempo de evolución. Si lleva menos de seis horas, se procede a la detorsión y fijación al escroto. Después de seis horas suele ser necesaria la orquiectomía, realizándose también la fijación del testículo contralateral. Repasa el diagnóstico diferencial del escroto agudo con la tabla correspondiente a esta pregunta.

P188

MIR 2001-2002

El reflujo vesicoureteral se produce como consecuencia de un fallo en el mecanismo valvular que impide el paso de orina desde la vejiga hacia el uréter. El daño renal viene determinado tanto por las altas presiones vesicales que se transmiten a los cálices renales como por la frecuencia de infecciones, que conducirán finalmente a la existencia de cicatrices renales. El reflujo se clasifica desde el punto de vista etiopatogénico en primario o secundario a diversas patologías tales como duplicación ureteral, divertículo, ureterocele, etc. Existe una clasificación en cinco grados, según la altura que alcanza el reflujo y el grado de dilatación pieloureteral, que tiene implicaciones pronósticas y terapéuticas.

P187 (MIR 01-02) Diagnóstico diferencial del escroto agudo

Desgloses comentados

515

Pediatría

Una vez diagnosticado, graduado y establecido si el reflujo es primario o secundario, conviene conocer el tamaño de los riñones y si muestran cicatrices de esclerosis, que se pueden apreciar ya a los cuatro o cinco meses de haber padecido una infección urinaria (respuesta 3 correcta); la ecografía permite valorar el tamaño renal, y la gammagrafía isotópica del parénquima renal es útil para descartar los tractos cicatrizales. La urografía i.v. y la tomografía también pueden cumplir estos propósitos. Finalmente, se medirá la presión arterial, así como el aclaramiento basal de creatinina.

El tratamiento del reflujo primario difiere según el grado: •• En los grados I y II se observa una elevada probabilidad de remisión espontánea. El tratamiento será expectante, con profilaxis antibiótica para mantener estéril la orina, y seguimiento trimestral con urocultivo. La ecografía renal anual o bienal también resulta útil para valorar el crecimiento renal. La gammagrafía renal con DMSA aplicada al concluir el período de tratamiento ayuda a seleccionar a los niños con tractos cicatrizales residuales que requieren seguimiento de la presión arterial. •• El seguimiento de los casos de reflujo grado III es muy parecido, pero se precisan gammagrafías periódicas del parénquima renal si se sospecha la formación de nuevas cicatrices. Más de la mitad de los niños con reflujo grado III acaba precisando cirugía. •• El reflujo grado IV y V requiere cirugía precoz, pues no es la norma la remisión espontánea. El tratamiento del reflujo secundario es el de la causa.

T6 P187

Hemato-oncología pediátrica

MIR 2005-2006

Pregunta de dificultad media acerca del neuroblastoma. El neuroblastoma es un tumor del sistema nervioso simpático que constituye el 10% de los tumores de la infancia. La patología oncológica más frecuente en la infancia son las leucemias representando el 30%. El tumor sólido más corriente es el tumor cerebral, y dentro de ellos, el astrocitoma. El tumor sólido extracraneal más frecuente es el neuroblastoma. Te ayudará saber el orden de frecuencia (de mayor a menos) de los cánceres en el niño: leucemias, cerebrales, linfomas, neuroblastoma, Wilms, tumores óseos, otros (retinoblastoma...). La edad media al diagnóstico son los dos años y el 90% son menores de cinco años (nuestra pregunta nos habla de un lactante de diez meses). La localización más común es el abdomen (70%), siendo el 50% de localización suprarrenal. P187 (MIR 05-06) Localización del neuroblastoma

Los resultados tras un tratamiento quirúrgico son excelentes. P188 (MIR 01-02) Grados del reflujo vesicoureteral

P188 (MIR 01-02) Clasificación del reflujo

La clínica depende de la localización. En la pregunta nos dan el caso típico de localización abdominal que es la más frecuente y que suele aparecer como una masa abdominal que sobrepasa la línea media (a diferencia del Wilms). Otras formas de presentación son:

516

Desgloses comentados

•• •• •• ••

Pediatría

Paravertebral: con clínica de compresión medular. Tórax: se suelen descubrir al hacer Rx de tórax por otras razones. Cabeza y cuello: pueden dar un síndrome de Horner. Nasofaringe (estesioneuroblastoma): suele manifestarse por epistaxis.

La localización más usual de las metástasis es por vía linfática y hemática a hígado, médula ósea y esqueleto. Consecuentemente, la opción 3 es correcta, puesto que ante todo diagnóstico de neuroblastoma se debe realizar un aspirado de médula ósea como parte del estudio de extensión al ser la médula una diana frecuente de metástasis. Para el diagnóstico se pueden usar: •• TC abdominal que visualiza el tumor como una masa de densidad mixta sólida y quística, además de calcificaciones en el 80%. •• Catecolaminas urinarias elevadas (ácido homovalínico, ácido vanilmandélico): éste es un dato específico de este tumor. •• Gammagrafía con MIBG (metayodobencilguanidina) marcada con isótopo radiactivo que capta catecolaminas. Por consiguiente, la opción 5 es falsa, dado que esta técnica se sigue usando. •• Biopsia, que permite el diagnóstico anatomopatológico. Los factores pronósticos, si bien difíciles de retener, son esenciales de cara al MIR, puesto que ya han sido objeto de preguntas en otras convocatorias. •• De buen pronóstico: -- Diagnóstico en menores de un año: por tanto, opción 1 falsa. -- Localización en cuello, mediastino posterior o pelvis. -- Vanilmandélico/Homovalínico mayor de 1. -- Aneuploidía del ADN (este aspecto ya fue preguntado en exámenes anteriores). •• De mal pronóstico: -- Localización abdominal. -- Enolasa sérica y/o ferritina elevadas. -- Amplificación del oncogén n-myc. -- Estadios III y IV. -- Deleción del brazo corto del cromosoma 1. El tratamiento debe ser quirúrgico siempre que sea posible, si es necesario administrando previamente quimioterapia en los tumores no resecables. En consecuencia, la opción 2 es falsa, puesto que la

presencia de metástasis hepáticas no contraindica el tratamiento quirúrgico. Por último, señalar que la hemihipertrofia es una malformación congénita que se asocia al tumor de Wilms o nefroblastoma y no al neuroblastoma.

P189

MIR 2001-2002

El tumor de Wilms es el tumor abdominal más frecuente en la infancia y el tumor renal más habitual. Puede presentarse aislado o asociado a diversas malformaciones congénitas. Las más usuales son las anomalías genitourinarias, seguidas de la hemihipertrofia y la aniridia. Se ha asociado a algunos síndromes: síndrome WARG (Wilms, aniridia, retraso mental y malformaciones genitourinarias), síndrome de Dennys Drash (Wilms, retinopatía y alteraciones genitourinarias) y el síndrome de Beckwith-Wiedemann. Se pueden encontrar deleciones del cromosoma 11. La edad media de diagnóstico de este tumor es hacia los tres años de edad (la edad media de diagnóstico del neuroblastoma es algo menor, hacia los dos años). La forma más frecuente de debut es como una masa abdominal asintomática (también es la forma más común de presentación del neuroblastoma). Otras manifestaciones clínicas son la hipertensión arterial (por compresión de la arteria renal por el tumor), hematuria, policitemia por producción de eritropoyetina y hemorragia intratumoral por traumatismos. La localización más frecuente de metástasis es el pulmón. La TC muestra una masa poco homogénea, con zonas de necrosis, hemorragia y calcificaciones focales pequeñas (aunque menos corrientes y menos acentuadas que en el neuroblastoma). Si bien la PAAF no ha demostrado aumentar el riesgo de diseminación del tumor en el trayecto de la punción, la biopsia no debe hacerse porque la rotura de la cápsula renal cambia el estadio. El tratamiento se establece con cirugía, radioterapia y/o quimioterapia, dependiendo del estadio. Te resultará más fácil estudiar el neuroblastoma y el nefroblastoma si los comparas. Para ello, puedes utilizar la tabla de la página anterior.

P189 (MIR 01-02) Neuroblastoma frente a nefroblastoma

Desgloses comentados

517

Pediatría T7

Enfermedades infecciosas

De acuerdo con los protocolos vigentes, sin la realización de pruebas complementarias no quedaría descartado el origen orgánico, por lo que la pregunta se anuló.

P186 P231

En esta pregunta se plantea el diagnóstico diferencial de enfermedades que cursan con fiebre y exantema. En este caso clínico debes buscar las pistas que te lleven al diagnóstico. Lo primero que debe llamar tu atención son las lesiones blanquecinas en cavidad oral, que no son otras que las famosas manchas de Koplik, que son patognomónicas de sarampión, siendo ésta la respuesta correcta. Recuerda que, el sarampión se presenta como un exantema morbiliforme que no blanquea a la presión y puede afectar a palmas y plantas, en el contexto de una infección respiratoria con fiebre alta. El hecho de que el niño tenga 7 meses apoya más el diagnóstico, ya que aún no ha recibido la vacuna de la triple vírica (cuya primera dosis se administra entre los 12 y los15 meses). Es importante también saber por qué no son las otras opciones. En primer lugar, la enfermedad de Kawasaki se diagnostica mediante criterios clínicos fáciles de recordar con la regla del ABCDEF. La F de Fiebre de al menos 5 días es el único criterio imprescindible y debe acompañarse de al menos 4 de los otros 5: A de Adenopatía (cervical mayor de 1,5 cm). B de Boca (afectación de labios, lengua y orofaringe). C de Conjuntivitis (bilateral no exudativa). D de Dedos (eritema, edema y posterior descamación de manos y pies). E de Exantema. Recuerda que estos síntomas no se pueden justificar por ninguna otra causa. En segundo lugar, no puede tratarse de una escarlatina aunque presente una faringoamigdalitis, ya que en menores de 3 años éstas son prácticamente en su totalidad víricas y recuerda que la causa de la escarlatina es un Streptoccocus pyogenes productor de toxina eritrógena. Fíjate además en que no ha respondido al tratamiento antibiótico. En relación al exantema alérgico por amoxicilina, se presentaría como una reacción de tipo urticarial. Por último, podrías pensar que es una mononucleosis por el exantema asociado al empleo de amoxicilina, que es muy característico. No obstante, deberían aportar otros datos como adenopatías múltiples o hepatoesplenomegalia.

P172

MIR 2009-2010

Este caso sugiere un dolor abdominal recidivante de origen funcional (no orgánico), que es frecuente en la infancia. Tratados de Pediatría, como el de Nelson, sostienen que el estudio de laboratorio no siempre es necesario en una primera visita. Sin embargo, también admite como manejo diagnóstico inicial la realización de un hemograma completo, con VSG, parásitos en heces y análisis de orina. Por ello, dado que son posibles varias actitudes, se trataría de una pregunta dudosa. Por otra parte, los protocolos de la Asociación Española de Pediatría sostienen que, ante un cuadro de dolor abdominal recurrente debemos cumplir ciertas condiciones para descartar un origen orgánico. Se deben descartar los siguientes signos de alarma: •• Clínicos (anamnesis y exploración). •• Por datos de laboratorio (alteraciones en el hemograma, marcadores inflamatorios elevados, anomalías del sedimento urinario y presencia de sangre oculta en heces).

518

MIR 2006-2007

MIR 2011-2012

Desgloses comentados

Con respecto de las vías de transmisión vertical del VIH neonatal, recuerda que la tasa de transmisión oscila entre 25-52% y que esta transmisión puede acontecer en tres momentos: vía transplacentaria (30-40% de los casos). Vía de transmisión durante el parto: vía más importante de transmisión maternofetal (60-70%). Dentro de las variables que facilitan la transmisión destaca la rotura de membranas superior a cuatro horas; otras son la prematuridad y el bajo peso al nacimiento. Vía postnatal: existe documentación de transmisión mediante la leche materna (es una de las pocas contraindicaciones de la lactancia materna, salvo en los países en vías de desarrollo).

P185

MIR 2005-2006

Pregunta fácil en forma de caso clínico típico de la enfermedad de Kawasaki. Esta pregunta expone, en forma de caso clínico, la historia de un paciente con enfermedad de Kawasaki. Esta entidad se diagnostica mediante el cumplimiento de sus criterios diagnósticos. Además de la fiebre, aquí aparecen cuatro: exantema, conjuntivitis bilateral no purulenta, cambios en la mucosa oral (eritema labial, lengua aframbuesada) y adenopatías. La enfermedad acostumbra a presentarse con fiebre alta de más de cinco días de evolución que no responde a antibióticos (el niño ha recibido tres dosis de azitromicina). Además de los cuatro criterios diagnósticos que aparecen en la pregunta, pueden aparecer alteraciones en las zonas periféricas de las extremidades, como edema y eritema en manos o pies, descamación de inicio periungueal, que suelen aparecer entre la primera y la tercera semana. La manifestación más significativa por su gravedad es la afectación cardíaca, que ocurre en un 10-40% de los casos. Se produce vasculitis coronaria en las dos primeras semanas con posterior formación de aneurismas en cuentas de rosario. Una regla mnemotécnica acerca de las características de la enfermedad de Kawasaki: ABCDEFG: adenopatías, boca (alteraciones mucosa orofaringe), conjuntivitis, descamación, exantema, fiebre y gammaglobulina en su tratamiento. Recuerda un dato muy novedoso (año 2005) acerca de la etiología de dicha enfermedad: se relaciona con el coronavirus New Haven que actuaría como un superantígeno. La clínica del exantema súbito es muy típica y fácil de reconocer en el MIR. Se produce en niños menores de tres años. Se caracteriza por fiebre muy alta con aparente buen estado general y sin foco durante tres a cinco días y al tercer o cuarto día la desaparición brusca de la fiebre coincidiendo con la aparición de un exantema que no afecta ni a cara ni a extremidades inferiores. Lo causa el VHH tipo 6.

Pediatría

P185 (MIR 05-06) Enfermedades exantemáticas y afines

La escarlatina es un cuadro brusco con fiebre y aspecto de enfermedad grave con una lengua en fresa blanca (recubierta por una capa blanquecina) primero y una lengua roja (por descamación aparece una lengua hiperémica). Las amígdalas aparecen cubiertas de un exudado blanco acompañadas de un enantema en el paladar blando. Asimismo, aparece un exantema que predomina en pliegues y que se palpa mejor que se ve (líneas de Pastia). En la cara existe un intenso eritema en frente y mejillas que respeta el triángulo nasogeniano (facies de Filatov). El cuadro está producido por el S. pyogenes. La rubéola se caracteriza por una fase prodrómica de catarro leve con fiebre moderada y conjuntivitis sin fotofobia. El signo más característico es la aparición 24 h antes del exantema de adenopatías retroauriculares,

cervicales posteriores y postoccipitales. El exantema es morbiliforme, se inicia en zonas retroauriculares y desde allí desciende centrífugamente afectando en 24 h a todo el cuerpo, respetando palmas y plantas. La mononucleosis infecciosa afecta típicamente a sujetos entre 15 y 25 años y la produce el VEB. El cuadro comienza con síntomas gripales que duran 7-14 días seguidos de un cuadro florido con fiebre alta, dolor faríngeo intenso, adenopatías de predomino cervical, hepatoesplenomegalia y rash cutáneo maculopapular (sobre todo, en pacientes tratados erróneamente con ampicilina al suponer que el cuadro de faringitis es de etiología bacteriana).

Desgloses comentados

519

Pediatría

P187

MIR 2004-2005

Las enfermedades exantemáticas son un tema relativamente importante. Lo esencial es saber hacer diagnóstico diferencial entre ellas. Este caso clínico presenta datos muy típicos, con lo que el diagnóstico no ofrece duda. Fiebre alta + exantema cuando ésta desaparece = EXANTEMA SÚBITO Es muy típico de esta enfermedad que el exantema respete la cara y extremidades inferiores. Recuerda que el agente causal es el HHV-6. También es característico de esta enfermedad la presencia de leucocitosis con neutrofilia en las primeras 24 horas y su asociación con las convulsiones febriles.

P172

MIR 2003-2004

Se trata de una pregunta fácil que se responde con información del Manual CTO. La enfermedad de Kawasaki o síndrome mucocutáneo ganglionar es de causa desconocida, pese a que se postula la hipótesis de una toxina que induciría un daño inmunitario endotelial (respuesta 3 verdadera). La enfermedad cursa con fiebre alta de más de cinco días de evolución, con mala respuesta a antibióticos (respuesta 2 verdadera) conjuntivitis bilateral, afectación de mucosa orofaríngea y de zonas periféricas de extremidades, fundamentalmente descamación de inicio periungueal, exantema polimorfo, linfadenopatías cervicales, etc. El diagnóstico es, fundamentalmente, clínico (respuesta 1 falsa). La complicación más grave es la afectación coronaria en las dos primeras semanas con aparición de aneurismas "en cuentas de rosario" (respuesta 4 verdadera) El tratamiento fundamental es gammaglobulina i.v. que previene la afectación coronaria y salicilatos a dosis altas (respuesta 5 verdadera).

T8 P181

Síndrome de la muerte súbita del lactante (SMSL)

MIR 2006-2007

Se consideran factores de riesgo biológico para el síndrome de la muerte súbita del lactante el sexo masculino, los antecedentes de prematuridad (dos o tres veces más riesgo), la falta de regulación térmica, la postura en prono para dormir, mayor frecuencia cardíaca en todas las fases del sueño-vigilia, defectos autonómicos, patrones respiratorios con alteraciones en la capacidad de autorreanimación con peor recuperación desde una hipoxemia. El QT prolongado también favorece la muerte súbita.

P166

MIR 2003-2004

Pregunta muy fácil, prácticamente de conocimiento popular. De la muerte súbita, principalmente, hay que saber que se relaciona con el decúbito prono y el tabaquismo materno.

520

Desgloses comentados

El SMSL se define como la muerte repentina e inesperada en un lactante por razones que no quedan claras ni después de la necropsia. Es la forma de muerte más común en el primer año de vida. La máxima incidencia es a los dos o tres meses de edad. Ocurren pocos casos antes de las dos semanas y después de los seis meses. Los varones tienen más riesgo que las mujeres. La incidencia es mayor en los meses fríos. Se ha supuesto que se produce durante el sueño, pues la mayoría de los casos se dan entre la medianoche y las nueve de la mañana. Se ha relacionado con múltiples factores biológicos y epidemiológicos, entre ellos la posición en decúbito prono y lateral (parece que porque interfieren con el control respiratorio, sobre todo en el momento del despertar).

T9 P259

Maltrato infantil

MIR 2008-2009

Una pregunta muy interesante. En ella, intentan confundirnos con infecciones exantemáticas como la varicela o la enfermedad de Kawasaki, pero si somos observadores es difícil que nos engañen. Se trata de un niño maltratado, y el dato principal lo tenemos en la piel, que nos permite descartar el resto de las opciones. •• Una sepsis con CID produciría lesiones diseminadas y malestar general. No justificaría una distribución como la descrita, y todavía menos un cefalohematoma parietal. •• La enfermedad de Kawasaki no produce lesiones purpúricas, o al menos sería muy infrecuente. •• Carece, por otra parte, de otros criterios sugestivos (conjuntivitis, adenopatías, etc.). •• El déficit congénito de antitrombina III produce un estado de trombofilia, es decir, tendencia a la hipercoagulabilidad, mientras que aquí están describiendo hematomas y equimosis. •• No existen lesiones vesiculosas (ni vesiculohemorrágicas) que pudieran hacernos pensar en la opción 4. Tampoco refieren fiebre, prurito, etc. Sin embargo, la caprichosa distribución de las lesiones y los bordes bien delimitados nos hacen pensar en una causa externa. Si, además, el niño no está bien nutrido y está muy agitado, habría que considerar un posible maltrato.

T10

Calendario vacunal infantil

P183

MIR 2010-2011

Pregunta de elevada dificultad sobre un tema bastante preguntado en el examen MIR: las inmunizaciones. La respuesta 1 es correcta, ya que en estudios recientes se ha concluido que la vacunación antineumocócica masiva de la población infantil menor de 2 años ha reducido significativamente la tasa de meningitis y enfermedad neumocócica invasiva (por los serotipos incluidos en la va-

Pediatría

cuna) en el conjunto de la población. Existen resultados convincentes de que la introducción de la PCV7 en la población infantil en los EE.UU. ha reducido la incidencia de enfermedad neumocócica invasiva en los niños y en los adultos, por lo que sería esperable que esto ocurriera en otros países. La respuesta 2 es falsa, ya que la PCV7 tiene su indicación en niños menores de 2 años, no en adultos. Las respuestas 3 y 4 son falsas porque hacen referencia a la vacuna polisacárida polivalente (PPV23) que se administra en niños mayores de 2 años y adultos con enfermedades crónicas, inmunodepresión, mayores de 65 años y personas institucionalizadas. Tiene una inmunidad poco duradera, por lo que se recomienda en ciertos casos la revacunación cada 3-5 años, y no ha demostrado reducir la incidencia de meningitis y enfermedad neumocócica invasiva. La mejora en las condiciones de vida y el mejor uso de los antibióticos (respuesta 5 incorrecta) han influido en la disminución de la incidencia de meningitis e infecciones invasivas por neumococo pero sin duda, lo más determinante ha sido la vacunación con PCV7 en niños menores de 2 años.

P185

MIR 2010-2011

Pregunta fácil sobre un tema muy repetido en el examen MIR, las contraindicaciones generales de las vacunas. La respuesta correcta es la 1 debido a que en cuadros con alteraciones inmunitarias importantes (inmunodeficiencias, tratamiento inmunosupresor intenso y prolongado, radioterapia…) están contraindicadas las vacunas de microorganismos vivos atenuados. Por tanto, de las vacunas que aparecen en el enunciado la única respuesta posible es la triple vírica. El resto son vacunas de toxoides o antígenos purificados, por lo que no suponen riesgo de infección para un inmunosuprimido.

Otros temas

P180

MIR 2007-2008

En ocasiones, los niños pequeños se introducen pequeños objetos en la nariz, en un intento normal de explorar sus propios cuerpos. Lo que se introducen puede ser muy variable: alimentos, semillas, borradores, juguetes. Es un cuadro relativamente frecuente y, sólo por la edad del niño, deberías haberte planteado seriamente responder la opción 4. Por otra parte, la clínica que nos describen encaja bastante bien: •• Olor fétido o secreción nasal sanguinolenta. •• Dificultad para respirar por la fosa nasal afectada. •• Sensación de ocupación en la nariz.

P241

MIR 2005-2006

Pregunta de dificultad media-alta que requiere el conocimiento de patologías maternas que pueden afectar transitoriamente al recién nacido, por lo que deberás conocerlas si vuelve a caer. Si nos fijamos bien descubrimos que todas las patologías que nos exponen tienen una etiopatogenia basada en anticuerpos maternos que pueden pasar al feto vía transplacentaria, radicando la dificultad en conocer en cuál de ellas no aparecen manifestaciones en el recién nacido. En todas pasan estas inmunoglobulinas, pero en una de ellas, la enfermedad celíaca en concreto, la aparición de síntomas no solamente depende de la presencia de éstas, sino que requiere otros factores determinantes como el factor genético y la presencia de gluten en la dieta (no hay gluten en la dieta de un recién nacido) y, asimismo, es necesario conocer que en esta enfermedad existe un intervalo libre de enfermedad que oscila entre tres y seis meses entre la introducción del gluten y la aparición de la enfermedad. Por todo eso, ésta sería la respuesta correcta. Con relación al resto de opciones que nos presentan: •• Si ahora consideramos la respuesta 1 sería falsa, pues en hasta el 15% de hijos de mujeres enfermas o futuras enfermas de miastenia gravis aparecen ya en el primer día postparto síntomas como ptosis palpebral, debilidad generalizada, dificultades de succión... que responden bien al tratamiento y que se recuperan, generalmente, el primer mes. •• En el caso del hipertiroidismo primario (respuesta 2) también sería falsa, pues se ha visto como existe paso transplacentario de inmunoglobulinas estimulantes del tiroides en caso de madres con enfermedad de Graves o tiroiditis de Hashimoto (siendo el propiltiouracilo el tratamiento de elección en la embarazada) que causa clínica de tirotoxicosis transitoria. Si la madre tomó drogas antitiroideas durante la gestación, los niños pueden estar asintomáticos al nacer y no presentar clínica hasta los cinco o diez días. •• Finalmente, las respuestas 3 y 4 son enfermedades ampollosas autoinmunes que pueden aparecer en la madre y que en un 5-10% de los casos los anticuerpos pasan la barrera placentaria y provocan durante unas semanas ampollas en el recién nacido (intraepidérmicas en el caso del pénfigo y subepidérmicas en el herpes gestationis), pues estos anticuerpos ocasionan acantolisis al atacar proteínas específicas en la piel y en las membranas mucosas.

P188

MIR 2004-2005

Pregunta muy difícil sobre un tema muy concreto de endocrinología pediátrica. No te preocupes en absoluto si la has fallado. El raquitismo resistente al calcitriol, también llamado tipo II, es de herencia autosómica recesiva. Se debe a una resistencia periférica a las acciones de la vitamina D, en relación con una anomalía de su receptor. Clínicamente, se manifiesta como un síndrome hipotónico y con signos y síntomas de raquitismo severo. Característicamente, a diferencia de lo que ocurre en otras carencias de vitamina D, asocia alopecia total.

Desgloses comentados

521

View more...

Comments

Copyright ©2017 KUPDF Inc.
SUPPORT KUPDF